You are on page 1of 41

MAY - 2023 1

QUESTION BANK

MAY
2023
QUESTIONS BASED ON IMPORTANT ARTICLES COVERED
FROM MULTIPLE SOURCES SUCH AS THE HINDU,
THE INDIAN EXPRESS AND THE PIB.

QUESTIONS IN LINE WITH UPSC SYLLABUS AND


QUESTION PAPER TRENDS.

ALSO INCLUDES ANSWER KEY AND


EXPLANATION FOR THE PRELIMS BASED
QUESTIONS.
MAY - 2023 2

UPSC PRELIMS PRACTICE QUESTIONS


1. Which of the following are the correct nutritional 4. Which of the following statements is incorrect
requirements for mid day meal scheme under the about Telecom Regulatory Authority of India (TRAI)?
National Food Security Act: (Level – Difficult) (Level-Difficult)
1. Protein quantity should be 15-20 gms A. The TRAI consists of a Chairperson, two whole-
2. Fat quantity should be 18-21 gms time members and two part-time members, all of
which are appointed by the Government of India.
3. Carbohydrates should be 70 gms
Choose the correct code: B. The Telecom Regulatory Authority of India (TRAI)
was established on 20th February, 1997 as an
A. Only one statement is wrong executive agency.
B. Two statements are wrong C. The recommendations of the TRAI are not
C. All statements are wrong binding upon the Central Government.
D. None of the statements is wrong D. The TRAI Act was amended in 2000 which
established a Telecommunications Dispute
2. Consider the following pairs: (Level-Medium) Settlement and Appellate Tribunal (TDSAT) to
take over the adjudicatory and disputes functions
from TRAI.
Mission name Country
1. Ganga Yemen 5. Consider the following pairs Town sometime
mentioned in news: (Level-Moderate) (PYQ-
2. Kaveri Sudan CSE-2018)
3. Devi Shakti Ukraine
Country Places
Which of these are matched incorrectly?
1 . Aleppo Syria
A. 1 & 3 only
B. 2 & 3 only 2. Kirkuk Yemen
C. 1 & 2 only 3. Mosul Palestine
D. All of the above 4. Mazar -i- Sharif Afghanistan

3. Consider the following statements with regards to


Which of the pairs given above are correctly
Shree Jagannath Puri temple:(Level-Easy)
matched?
1. Jagannath Puri temple is called ‘Yamanika Tirtha’
A. 1 and 2
where, according to the Hindu beliefs, the power
of ‘Yama’, the god of death has been nullified in B. 1 and 4
Puri due to the presence of Lord Jagannath. C. 2 and3
2. This temple was called the “White Pagoda” and D. 3 and 4
is a part of Char Dham pilgrimages (Badrinath,
Dwaraka, Puri, Rameswaram). 6. How many of the following statements is/are
3. The temple is believed to be constructed in the true regarding “Mission Karmayogi”? (Level – Easy)
12th century by King Anatavarman Chodaganga 1. It is a scheme to commemorate the Yogic
Deva of the Western Ganga Dynasty. heritage of India.
Choose the correct statements: 2. It is designed for the government servants of the
A. 1 & 2 only country.
B. 2 & 3 only 3. iGOT Karmashala is one of the initiatives under
C. 1 & 3 only this scheme.
D. All of the above Options:
A. Only 1 statement
B. Only 2 statements
C. All 3 statements
D. None of the statements
MAY - 2023 3
7. Identify the correct statements: (Level – Difficult) 11. A and B are 2 farmers. A holds land of less than 2
1. Patents are given for novel inventions and hectares while B’s land is 5 hectares. Their neighbour
discoveries. C does not own land but has a herd of buffaloes
whose milk he sells to earn a livelihood. A, B, and C
2. Indian Patents Act 2005 provides for 20 years of
live in a coastal district and procure their daily fish
protection.
needs from D who is a fisherwoman.
Options:
A. 1 only
Who among the above will be eligible for a Kisan
B. 2 only Credit Card? (Level – Difficult)
C. Both 1 and 2 A. C only
D. Neither 1 nor 2 B. A and C only
C. B only
8. Which of the following statements is/are correct? D. A, B, C and D
(Level – Easy)
1. The Malthusian Population theory states that the
12. Which of the following statements are true regarding
population increases at geometric progression
the POSH Act, 2013? (Level – Moderate)
while the resources grow at arithmetic progression.
1. The Vishakha Guidelines given by the Supreme
2. The total fertility rate is the total number of children
Court were a precursor to this law.
a woman will have during her entire lifetime.
2. Every employer should establish an Internal
3. The replacement TFR is 2.1.
Complaints Committee.
Options:
3. The ICC should have at least one woman member.
A. 1 and 2
4. For unorganized sectors, the State must direct
B. 2 and 3 the district officers to establish a Local Complaints
C. 1 and 3 Committee.
D. 1, 2 and 3 Options:
A. 1, 2, 3 and 4
9. Which of the following statements is/are true? (Level B. 1, 2 and 4
– Difficult) C. 3 and 4
1. The Ministry of Jal Shakti takes up the Swachh D. 1 and 3
Sarvekshan in both Urban and Rural areas.
2. ODF ratings can be provided if at any point in the
13. The Yanomani Area, in news for illegal mining and
day, not a single person is found defecating in
clashes between miners and indigenous people, is
public in that area, and untreated faecal matter is
located in which country? (Level – Difficult)
not dumped in the water bodies.
A. Brazil
3. ODF + ratings, apart from zero people defecating
in the open at any point of the day, require that all B. Indonesia
the community and public toilets properly function C. Vietnam
in that area. D. Philippines
Options:
A. 1 only 14. How many of the following bodies is/are a part of the
B. 1, 2 and 3 World Bank Group? (Level – Easy)
C. 3 only 1. Asian Development Bank
D. 2 and 3 only 2. Asian Infrastructure Investment Bank
3. International Bank for Reconstruction and
10. The substitution of steel for wooden ploughs in Development
agricultural production is an example of: (Level – 4. Multilateral Guarantee Agency
Moderate) [PYQ 2015] 5. International Centre for the Settlement of
A. Labour-augmenting technological progress Investment Disputes
B. Capital-augmenting technological progress Options:
C. Capital-reducing technological progress A. Only 2 bodies
D. None of the above B. Only 3 bodies
C. Only 4 bodies
D. All 5 bodies
MAY - 2023 4
15. Westerlies in the southern hemisphere are stronger 20. Which one of the following statements does not
and persistent than in the northern hemisphere. apply to the system of Subsidiary Alliance introduced
Why? (Level – Easy) [PYQ 2011] by Lord Wellesley?​(Level-Easy)
1. Southern hemisphere has less landmass as A. To maintain a large standing army at other’s
compared to the northern hemisphere. expense​
2. Coriolis force is higher in the southern hemisphere B. To keep India safe from Napoleonic danger​
as compared to the northern hemisphere. C. To secure a fixed income for the Company​
Which of the statements given above is/are correct? D. To establish British paramountcy over the Indian
A. 1 only States
B. 2 only
C. Both 1 and 2 21. Which of the following are the projects developed
D. Neither 1 nor 2 by the Border Roads Organisation(BRO)? (Level-
Difficult)
16. Which of the following statements is/are true? (Level- 1. Beacon
Difficult) 2. Dantak
1. The exchange rates in India are determined by the 3. Sampark
Reserve Bank of India. 4. Vartak
2. Currency devaluation can be done only by the 5. Vijayak
government, while currency depreciation is a Options:
result of market forces.
A. 1, 2 and 3 only
Options:
B. 1, 4 and 5 only
A. 1 only
C. 2, 3, 4 and 5 only
B. 2 only
D. 1, 2, 3, 4 and 5
C. Both 1 and 2
D. Neither 1 nor 2 22. Which article in the Indian Constitution allows
the Union to protect every State against external
17. Which of the following statements is/are true? (Level- aggression and internal disturbance and to ensure
Medium) that the government of every State is carried on in
1. Type 1 diabetes is caused due to auto-immune accordance with the provisions of this Constitution?
reactions. (Level-Medium)
2. Type 2 diabetes is caused only due to insulin A. Article 351
deficiency in the body. B. Article 355
3. Type 1 diabetes is more common in India. C. Article 358
Options: D. Article 361
A. 1 only
B. 1 and 2 23. Which of the following statements is/are correct with
C. 2 and 3 respect to rare diseases?(Level-Medium)
D. 1, 2 and 3 1. All the rare diseases are genetic and hence affect
a large number of children.
18. Identify the correct statement: (Level-Medium) 2. According to the National Policy of Rare Diseases,
Group 1 diseases are those requiring long-term or
1. The Central government has started the lifelong treatment.
“Aspirational Blocks” program on the lines of the
“Aspirational Districts” program. Options:
2. It has also initiated the “Aspirational Cities” program A. 1 only
to make Indian cities more liveable. B. 2 only
Options: C. Both
A. 1 only D. None
B. 2 only
C. Both 1 and 2
D. Neither 1 nor 2

19. Which of the following countries do not border the


Mediterranean Sea? (Level-Easy)
A. Israel
B. Jordon
C. Syria
D. Tunisia
MAY - 2023 5
24. Consider the following statements: (Level-Difficult) 28. Telegraph Memorial made of grey Aberdeen granite
1. Loyumba Shinyen, the written constitution of and standing on a sandstone base, was unveiled by
Kangleipak was formally developed by Pitambar Lord Curzon. It is dedicated to the efforts and sacrifice
Charairongba. of telegraphers during the (Level – Moderate)
2. Manipur learned the art of making gunpowder A. Paika Rebellion of 1817
from the Chinese merchants. B. Revolt of 1857
3. At the conclusion of the First Anglo-Burmese War, C. Vellore Mutiny
Manipur became a British protectorate. D. Sannyasi rebellion
Which of the statements given above is/are correct?
A. 1 only 29. Consider the following Pairs: (Level – Difficult)
B. 2 and 3 only
C. 1 and 3 only Initiatives State/UT
D. 1, 2 and 3 1. Chandauli Black Rice Initiative Uttar Pradesh
2. Kalika Chetarike Karnataka
25. Satellites used for telecommunication relay are kept
in a geostationary orbit. A satellite is said to be in 3. Santhe Kaushalkar Telangana
such an orbit when: (Level-Difficult) (PYQ-CSE-201)
4. SARTHI & SAKHI -Mental Maharashtra
1. The orbit is geosynchronous. Health Helpline
2. The orbit is circular.
3. The orbit lies in the plane of the Earth’s equator. How many pairs given above are correctly matched?
4. The orbit is at an altitude of 22,236 km. A. Only one pair
Select the correct answer using the codes given B. Only two pairs
below: C. Only three pairs
A. 1, 2 and 3 only D. All four pairs
B. 1, 3 and 4 only
C. 2 and 4 only 30. In India, if a religious sect/community is given the
D. 1, 2, 3 and 4 status of a national minority, what special advantages
it is entitled to? (Level – Easy) PYQ (2011)
1. It can establish and administer exclusive
26. Consider the following statements w.r.t Space
educational institutions.
science and Technology AwaReness Training (START)
programme: (Level – Easy) 2. The President of India automatically nominates a
representative of the community to Lok Sabha.
1. It is a sponsored programme organised by ISRO
for School Children to impart basic knowledge on 3. It can derive benefits from the Prime Minister’s
Space Technology. 15-Point Programme.
2. The programme will cover various domains Which of the statements given above is/are correct?
of space science, including Astronomy & A. 1 only
Astrophysics, Heliophysics & Sun-Earth interaction, B. 2 and 3 only
Instrumentation, and Aeronomy. C. 1 and 3 only
Which of the statements given above is/are correct? D. 1, 2 and 3
A. 1 only
B. 2 only 31. Wushu, a martial art form, originated in which
C. Both country? (Level – Easy)
D. None A. Japan
B. China
27. Which of the following laws are associated with a C. Vietnam
‘Shoot At Sight’ Order? (Level – Difficult) D. Korea
1. Armed Forces (Special Powers) Act (AFSPA)
2. Code of Criminal Procedure (CrPC) 32. Which of the following statements are true regarding
3. Indian Penal Code (IPC) the I2U2 grouping? (Level – Moderate)
4. Prison Act 1. It was started in the year 2021.
5. Probation of Offenders Act 2. Apart from India, the other members include the
Options: USA, UK, and Israel.
A. 1, 2 and 3 only 3. It is also known as the West Asian Quad.
B. 2, 3 and 4 only Options:
C. 1, 4 and 5 only A. 1 only
D. 1, 2, 3, 4, and 5 B. 2 and 3
C. 3 only
D. 1 and 3
MAY - 2023 6
33. Identify the correct statements: (Level – Easy) Options:
1. Both Nagara and Dravidian temples have an A. Only 1 statement
element called Shikhara. B. Only 2 statements
2. Gopurams are a prominent feature of the Dravidian C. All 3 statements
Temples.
D. None of the statements
Options:
A. 1 only 38. Which of these statements are true? (Level – Difficult)
B. 2 only 1. Rabindranath Tagore was the first non-European to
C. Both 1 and 2 win the Nobel Prize in literature.
D. Neither 1 nor 2 2. He gave up his knighthood to protest the Rowlatt
Act (The Anarchical and Revolutionary Crimes Act)
34. How many of the following countries are a member of 1919.
of the Arab League? (Level – Difficult) 3. He is the author of a collection of poems “Manasi”.
1. Syria Options:
2. Egypt A. 1 and 2
3. Oman B. 2 and 3
4. Israel C. 1 and 3
5. Tunisia D. 1, 2 and 3
Options:
A. Only 2 countries 39. The Enforcement Directorate is a nodal agency for
B. Only 3 countries which of these following Acts of Parliament? (Level –
Difficult)
C. Only 4 countries
1. The Prevention of Money Laundering Act, 2002
D. All 5 countries
(PMLA)
2. The Foreign Exchange Management Act, 1999
35. On which of the following can you find the Bureau (FEMA)
of Energy Efficiency Star Label? (Level – Moderate)
[PYQ 2016] 3. The Fugitive Economic Offenders Act, 2018 (FEOA)
Options:
1. Ceiling fans
2. Electric geysers A. 1 only
3. Tubular fluorescent lamps B. 1 and 2 only
Select the correct answer using the code given below. C. 2 only
A. 1 and 2 only D. 1, 2 and 3
B. 3 only
40. Consider the following statements: (Level – Easy)
C. 2 and 3 only
[PYQ 2016]
D. 1, 2 and 3
1. New Development Bank has been set up by APEC.
2. The headquarters of New Development Bank is in
36. Identify the correct statements: (Level – Moderate) Shanghai.
1. Mission LiFE was introduced by India at UNFCCC’s Which of the statements given above is/are correct?
COP 26.
A. 1 only
2. It has a dual objective of achieving the SDGs and
combating climate change.
B. 2 only
3. It focuses on biodiversity conservation. C. Both 1 and 2
Options: D. Neither 1 nor 2
A. 1 and 2
B. 2 and 3
C. 1 and 3
D. 1, 2 and 3

37. How many of the following statements is/are true


regarding the cyclones in India? (Level – Moderate)
1. India has 2 cyclone seasons.
2. Cyclones are more frequent in the Bay of Bengal
compared to the Arabian Sea due to its higher
surface temperature.
3. Cyclone Mocha, the first cyclonic storm of 2023,
has been named by Yemen.
MAY - 2023 7
41. Consider the following statements with regard to the D. The NCLAT was constituted under Section 410 of
Laser Interferometer Gravitational-Wave Observatory the Companies Act, 2013 to hear appeals against
(LIGO) project: (Level – Easy) the orders of the National Company Law Tribunal
1. The Indian LIGO would have two perpendicularly (NCLT).
placed 4-km-long vacuum chambers, that
constitute the most sensitive interferometers in the 45. Which of the following statements are correct
world. regarding the general difference between plant and
2. It will be located in the Hingoli district of animal cells? (Level – Easy) [PYQ - 2020]
Maharashtra, about 450 km east of Mumbai. 1. Plant cells have cellulose cell walls whilst animal
3. The LIGO in the US first detected gravitational cells do not.
waves in 2013, which led to a Nobel Prize in Physics 2. Plant cells do not have plasma membranes unlike
in 2017. animal cells which do.
Choose the correct code:
3. Mature plant cell has one large vacuole whilst an
A. Only one statement is wrong animal cell has many small vacuoles.
B. Two statements are wrong Select the correct answer using the code given below:
C. All statements are wrong A. 1 and 2 only
D. None of the statements is wrong B. 2 and 3 only
C. 1 and 3 only
42. Consider the following statements about Section 144
of the CrPC: (Level – Easy) D. 1, 2 and 3
1. This law empowers the magistrate of any state or
union territory in India to pass an order prohibiting 46. Consider the following statements about Biodiversity
the gathering of four or more people in a specified Hotspots:(Level – Moderate)
area. 1. The term was coined by UNESCO.
2. According to the order under this section, there 2. There are 36 Biodiversity hotspots globally,
shall be no movement of the public but the covering 2.5% of Earth’s land.
educational institutions may remain open. 3. 4 Biodiversity Hotspots are located in India.
3. It doesn’t empower the authorities to block internet How many of the above statements are correct?
access in the region. A. Only 1 statement
Which of these are incorrect? B. Only 2 statements
A. 1 & 3 only C. All 3 statements
B. 2 & 3 only D. None of the statements
C. 1 & 2 only
D. All of the above 47. Which of the following pairs are correctly matched
with regards to the river, and the country boundary it
43. Which of the following best defines the phrase marks? (Level – Difficult)
‘persona non grata’? (Level–Moderate)
1. Rio Grande: USA and Mexico
A. It refers to a diplomat or foreign person whose
2. Orange River: Namibia and South Africa
entering or remaining in a certain country has been
prohibited by that country 3. Amur River: China and Russia
B. It refers to a diplomat whose immunity has been Options:
taken away due to gross violation of a law A. 1 and 2
C. It refers to a diplomat or foreign person who B. 2 and 3
doesn’t need a visa to visit the country C. 1 and 3
D. It refers to a diplomat or foreign person who acts D. 1, 2 and 3
as the ambassador of a nation in the absence of
the full-time ambassador
48. Which of the following can be the impact of waiving
44. Which of the following statements is incorrect? (Level off import duties by the government? (Level – Easy)
– Easy) 1. It can reduce inflation in the domestic market.
A. NCLT (National Company Law Tribunal) is a quasi- 2. It can increase the spending capacity of the
judicial body that adjudicates issues relating to citizens.
companies. 3. It can disturb the country’s balance of payment.
B. Any person aggrieved by any order of the NCLAT 4. It can give a boost to domestic industries.
(National Company Law Appellate Tribunal) may Options:
file an appeal to the High Court.
A. 1, 2 and 3
C. NCLAT is also the appellate tribunal for orders
B. 2, 3 and 4
passed by the NCLT(s) under Section 61 of the
Insolvency and Bankruptcy Code (IBC), 2016, C. 1, 3 and 4
and for orders passed by the Insolvency and D. 1, 2, 3 and 4
Bankruptcy Board of India (IBBI) under Sections
202 and 211 of the IBC.
MAY - 2023 8
49. Identify the correct statements: (Level – Difficult) 54. Which of the following statements is incorrect with
1. The Cham lama dance is associated with Tibetan respect to LIBOR? (Level-Difficult)
Buddhism. A. It is the global reference rate for unsecured short-
2. In India, it is performed only in Ladakh in India. term borrowing in the interbank market.
3. Special masks and headgear are adorned by the B. The five currencies for which LIBOR is computed
performers in this dance. are the Swiss franc, euro, pound sterling, Japanese
Options: yen and US dollar.
A. 1 only C. LIBOR is administered by the Intercontinental
Exchange or ICE.
B. 2 only
D. The United Kingdom financial regulator that
C. 1 and 3 oversees LIBOR has announced that they’ll
D. 2 and 3 discontinue the index by 2030.

50. A person stood alone in a desert on a dark night and


55. They use living organisms as catalysts to generate
wanted to reach his village which was situated 5 km
electricity from certain substrates.
east of the point where he was standing. He had no
instruments to find the direction but he located the 1. They use a variety of inorganic materials as
polestar. The most convenient way now to reach his substrates.
village is to walk in the: (Level – Moderate) [PYQ - 2. They can be installed in waste water, treatment
2012] plants to cleanse water and produce electricity.
A. Direction facing the polestar Which of the statements given above is/are correct?
B. Direction opposite to the polestar A. 1 only
C. Direction keeping the polestar to his left B. 2 and 3 only
D. Direction keeping the polestar to his right C. 1 and 3 only
D. 1, 2 and 3
51. ‘Fisher effect’ describes the relationship between:
(Level-Difficult)
56. With respect to Khooni Darwaza, which of the
A. Inflation and Unemployment following statements is/are correct? (Level – Difficult)
B. Income Inequality and Economic Growth 1. Khooni Darwaza is a gate built during the reign of
C. Inflation and Interest Rate Sher Shah Suri in the 16th century.
D. Tax Rate and Tax Revenue 2. The architecture of Khooni Darwaza is a blend of
Mughal and Nagara Styles.
52. With respect to Default Bail, which of the following 3. The gate gets its name from the killings that took
statements is/are correct? (Level-Difficult) place there during the Indian Rebellion of 1857.
1. According to Section 187(2) CrPC, an accused is Options:
entitled to default bail if the investigating agency A. 1 only
failed to file a final charge sheet within 60 days
from the date of remand. B. 2 and 3 only
2. It is given to bailable offences only. C. 1 and 3 only
Options: D. 1, 2 and 3
1. 1 only
57. Who are the Persons entitled to vote by post at an
2. 2 only
election in an assembly constituency? (Level –
3. Both Moderate)
4. None 1. Special Voters
53. Consider the following statements with respect to 2. Service Voters
the Prevention of Sexual Harassment (PoSH) Act, 3. Voters on election duty
2013: (Level-Medium) 4. Electors subjected to preventive detention
1. Every employer must constitute an Internal Options:
Complaints Committee (ICC) at each office or A. 1, 2 and 3 only
branch that has 20 or more employees.
B. 2, 3 and 4 only
2. The Act protects the rights of all women who are C. 1, 3 and 4 only
working or visiting any workplace, in any capacity.
D. 1, 2, 3 and 4
3. If the woman cannot complain because of “physical
or mental incapacity or death or otherwise”, her
legal heir may do so.
Which of the following statements is/are correct?
A. 1 and 2 only
B. 2 and 3 only
C. 1 and 3 only
D. 1, 2 and 3
MAY - 2023 9
58. Which of the following states are recognized by 63. Which of the following statements are true regarding
Nuclear Non-Proliferation Treaty (NPT) as Nuclear Wholesale Price Index? (Level – Moderate)
Weapon States (NWS)? (Level – Easy) 1. It captures the change in the prices of goods and
1. France services before they enter the retail market.
2. Russia 2. It is used by the Reserve Bank of India for inflation
3. Iran targeting.
4. China Options:
5. Israel A. 1 only
Options: B. 2 only
A. 1, 2 and 3 only C. Both 1 and 2
B. 2, 3, 4 and 5 only D. Neither 1 nor 2
C. 1, 2 and 4 only
D. 1, 4 and 5 only 64. How many of the following statements are true
regarding the Mandovi River? (Level – Difficult)
59. Consider the following statements: (Level – Moderate) 1. It originates in the state of Karnataka.
1. The planet is sometimes called “The Jewel of the 2. It flows through 3 states in India before entering
Solar System.” the Bay of Bengal.
2. It is the least dense planet in the Solar System. 3. Salim Ali Bird Sanctuary is located on the banks of
The above statements best describe: this river.
A. Jupiter Options:
B. Saturn A. Only 1 statement
C. Uranus B. Only 2 statements
D. Neptune C. All 3 statements
D. None of the statements
60. A layer in the Earth’s atmosphere called Ionosphere
facilitates radio communication. Why? (Level – 65. With reference to the Genetically Modified mustard
Moderate) PYQ (2011) (GM mustard) developed in India,consider the
1. The presence of ozone causes the reflection of following statements: (Level – Difficult) [PYQ 2018]
radio waves to Earth. 1. GM mustard has the genes of a soil bacterium that
2. Radio waves have a very long wavelength. give the plant the property of pest-resistance to a
Which of the statements given above is/are correct? wide variety of pests.
A. 1 only 2. GM mustard has the genes that allow the plant
cross-pollination and hybridization.
B. 2 only
3. GM mustard has been developed jointly by the
C. Both 1 and 2
IARI and Punjab Agricultural University.
D. Neither 1 nor 2
Which of the following statements given are correct?
61. “Just Transition”, a term in the news these days, is A. 1 and 3 only
related to? (Level – Easy) B. 2 only
A. Transition from remote to office working post- C. 2 and 3 only
pandemic D. 1, 2 and 3 only
B. Tackling dangers of AI replacing human jobs
C. Transition from high to low carbon energy 66. Which of the following statements is/are true? (Level
generation – Difficult)
D. Indian efforts of import substitution for EV batteries 1. Sodium-ion batteries are a less efficient commonly
available alternative to lithium-ion batteries.
62. Which of the following activities will be a violation of 2. India has higher deposits of sodium than lithium.
the Model Code of Conduct? (Level – Moderate) 3. India imports more than 70% of its Lithium-ion
1. Ministers merging official visits and election batteries from China.
campaign visits. Options:
2. Having an impromptu gathering at a freely A. 1 and 2 only
available village ground without informing the
B. 2 and 3 only
administration.
C. 1 and 3 only
3. A political party member urging the voters to go
and cast their vote in maximum numbers. D. 1, 2 and 3
Options:
A. 1 and 2
B. 2 and 3
C. 1 and 3
D. 1, 2 and 3
MAY - 2023 10
67. India experiences pre-monsoon showers before the 71. Identify the correct statements regarding
monsoon season arrives in the month of June. How Cardiovascular Diseases: (Level – Moderate)
many of the pre-monsoon showers are correctly 1. It is the leading cause of death in the world.
matched with the state(s) they affect? (Level –
2. India Hypertension Management Project was
Moderate)
introduced in 2017 to reduce both deaths and
1. Cherry Blossom Showers: Meghalaya disabilities due to CVD.
2. Bordoisila: Assam 3. MoH&FW has recently introduced a 75/25 initiative
3. Mango Showers: Kerala solely to focus on patients with hypertension.
4. Coffee Showers: Karnataka Options:
Options: A. 1 and 2
A. Only 1 pair B. 2 and 3
B. Only 2 pairs C. 1 and 3
C. Only 3 pairs D. 1, 2 and 3
D. All 4 pairs
72. Which of the following statements is/are true? (Level
68. Which of the following statements is/are true? (Level – Difficult)
– Moderate) 1. Greater Flamingos are endemic to India.
1. India conducted its 1st nuclear test, codenamed 2. Greater Flamingos are the only variety of flamingos
“Operation Shakti” in 1974. in India.
2. It was an underwater test, conducted off the shore 3. They are mostly concentrated in the Eastern parts
of Maharashtra in the Arabian Sea. of India.
3. India is not a signatory to the Nuclear Non- 4. In the IUCN Red list, they are under the “Least
Proliferation Treaty. Concern” category.
Options: Options:
A. 1 only A. 1 and 2 only
B. 1 and 2 only B. 2 and 3 only
C. 3 only C. 4 only
D. 1 and 3 only D. 1, 3 and 4 only

69. Which of the following statements is/are true 73. Which of the following statements is/are true
regarding Endosulfan? (Level – Moderate) regarding the Nutrient Based Subsidy Scheme?
1. It is an insecticide that is associated with medical (Level – Easy)
conditions like neurotoxicity. 1. It is implemented by the Ministry of Agriculture and
2. It is covered under both the Stockholm and the Farmers’ Welfare.
Rotterdam Conventions. 2. It provides subsidies for all the 3 macronutrient
3. It has been banned in India. fertilizers (NPK).
Options: Options:
A. 1 and 2 A. 1 only
B. 2 and 3 B. 2 only
C. 1 and 3 C. Both 1 and 2
D. 1, 2 and 3 D. Neither 1 nor 2

70. The 1929 Session of Indian National Congress is of 74. Identify the animal by taking a look at the following
significance in the history of the Freedom Movement statements: (Level – Moderate)
because: (Level – Easy) [PYQ 2014] 1. This is an elusive species native to Central and
A. Attainment of self-government was declared as South Asia.
the objective of the Congress 2. In India, they are found in both Western and Eastern
B. Attainment of Poorna Swaraj was adopted as the Himalayas.
goal of the Congress 3. It is included in Appendix 1 of the CITES.
C. Non-Cooperation Movement was launched Options:
D. Decision to participate in the Round Table A. Sangai deer
Conference in London was taken B. Red Panda
C. Brown Bear
D. Snow Leopard
MAY - 2023 11
75. Which of the following is not included in the assets 80. Under the Constitution of India, which one of the
of a commercial bank in India? (Level – Easy) [PYQ following is not a fundamental duty? (Level – Easy)
2019] (PYQ- 2011)
A. Advances A. To vote in public elections
B. Deposits B. To develop the scientific temper
C. Investments C. To safeguard public property
D. Money at call and short notice D. To abide by the Constitution and respect its ideals

76. Which of the following differences between Severe 81. With respect to Alternative Investment Funds (AIF),
Acute Malnutrition (SAM) and Moderate Acute which of the following statements is/are correct?
Malnutrition (MAM) is/are correct? (Level – Moderate) (Level-Medium)
1. MAM may require only short-term treatment. The 1. AIF is any fund established in India which is a
duration of treatment for SAM is typically longer. privately pooled investment vehicle that collects
2. MAM can often be managed with simpler funds from sophisticated investors, both Indian or
interventions whereas SAM requires specialized foreign, for investing.
medical care. 2. AIF includes funds covered under Indian market
Options: regulator SEBI’s mutual fund regulations.
A. 1 only 3. Real estate funds, debt funds, private equity funds,
B. 2 only funds for distressed assets, etc. are registered as
Category III AIFs.
C. Both
Options:
D. None
A. 1 only
B. 2 and 3 only
77. Consider the following statements: (Level – Moderate)
C. 1 and 3 only
1. Pir Panjal is the largest range of the Lesser
Himalayas. D. 1, 2 and 3
2. The Pir Panjal Pass connects Srinagar and Leh.
82. Consider the following statements: (Level-Easy)
3. The renowned Galyat mountains are located in the
Pir Panjal Range. 1. Angel tax is levied on the capital raised by a listed
Which of the statements given above is/are correct? company by selling shares to investors above the
fair market value.
A. 1 and 2 only
2. An entity will be considered a startup up to 10 years
B. 2 and 3 only from the date of its incorporation and registration,
C. 1 and 3 only and an entity will continue to be recognised as a
D. 1, 2 and 3 startup if its turnover for any of the financial years
since incorporation and registration does not
78. Which of the following statements is/are correct? exceed ₹100 crore.
(Level – Difficult) Which of the statements given above is/are correct?
1. Political parties are provided free airtime on public A. 1 only
broadcasters during elections. This scheme is B. 2 only
available to national and recognised State parties. C. Both
2. The parties are allocated a base time and additional D. None
slots based on performance in previous polls.
3. The facility to provide free airtime for political 83. Herschel Island, that was seen in the news recently,
parties during elections was given a statutory is a part of which country? (Level-Difficult)
basis through the 2015 amendment to the
Representation of People Act, 1951.
A. Argentina
Options: B. Australia
A. 1 only C. Canada
B. 3 only D. Mexico
C. 2 and 3 only
D. 1 and 2 only

79. Lumpy Skin Disease (LSD) is a devastating disease of


cattle and buffalo caused by (Level – Easy)
A. Bacteria
B. Fungi
C. Protozoa
D. Virus
MAY - 2023 12
84. Which of the following statements is/are incorrect? 88. The third schedule of the Indian Constitution
(Level-Easy) contains the forms of oath or affirmation for which of
1. The right to protest is recognised as a fundamental the following constitutional offices? (Level – Easy)
right. 1. President of India
2. The right to peaceful protest is included in the 2. Union Ministers of India
Universal Declaration of Human Rights of the 3. Supreme Court Judges
United Nations.
4. Comptroller and Auditor General
Options:
5. Governors
A. 1 only
6. State Minister
B. 2 only Options:
C. Both A. 1, 2, 3 and 6 only
D. None B. 3, 4, 5 and 6 only
C. 2, 3, 4 and 6 only
85. Two important rivers - one with its source in Jharkhand
(and known by a different name in Odisha), and
D. 1, 3, 4 and 5 only
another, with its source in Odisha - merge at a place
only a short distance from the coast of Bay of Bengal 89. Consider the following statements with respect to
before flowing into the sea. This is an important site Deputy Chief Minister: (Level – Moderate)
of wildlife and biodiversity and a protected area. 1. A deputy Chief Minister is not a constitutional post
Which one of the following could be this? (Level- and is equivalent to the rank of a cabinet minister.
Difficult) (PYQ-CSE-2011) 2. The official files that are meant for CM are routed
A. Bhitarkanika through the deputy CM.
B. Chandipur-on-sea 3. He/she is appointed by the Chief Minister.
C. Gopalpur-on-sea Which of the statements given above is/are correct?
D. Simlipal A. 1 only
B. 2 and 3 only
86. Which of the following statements is/are correct? C. 1 and 3 only
(Level – Easy) D. 1, 2 and 3
1. Hiroshima was the first city in the world that was hit
by a nuclear weapon.
90. With reference to India, consider the following
2. Potsdam Declaration was issued by the United Central Acts: (Level – Moderate) PYQ (2011)
States, Great Britain, and China in 1945, calling for
1. Import and Export (Control) Act, 1947
the unconditional surrender of Japan.
Options: 2. Mining and Mineral Development (Regulation) Act,
1957
A. 1 only
3. Customs Act, 1962
B. 2 only
4. Indian Forest Act, 1927
C. Both
Which of the above Acts have relevance to/bearing
D. None on the biodiversity conservation in the country?
A. 1 and 3 only
87. With respect to Scheduled Castes and the Scheduled B. 2, 3 and 4 only
Tribes (Prevention of Atrocities) Amendment Act,
2018, which of the following statements is/are C. 1, 2, 3 and 4
correct? (Level – Difficult) D. None of the above Acts
1. Preliminary inquiry should be conducted before
registration of an FIR. 91. How many of the following philosophy and founder
2. Persons accused of committing an offence under pairs are correctly matched? (Level – Moderate)
the 1989 Act will not be eligible for anticipatory 1. Yoga: Patanjali
bail. 2. Vaisheshika: Jaimini
Options: 3. Samkhya: Kapila
A. 1 only 4. Nyaya: Gouthama
B. 2 only Options:
C. Both A. Only 1 pair
D. None B. Only 2 pairs
C. Only 3 pairs
D. All 4 pairs
MAY - 2023 13
92. Who among the following is/are eligible for an OCI 96. How many of the following statements is/are true?
Card? (Level – Difficult) (Level – Moderate)
1. A person whose mother is an Indian citizen, but 1. The Krishna River originates in Mahabaleshwar in
whose father is a citizen of the USA. Maharashtra.
2. An 80-year-old lady whose mother and father were 2. It is the 3rd longest river in India.
Indian Citizens, but they left India in 1946. 3. It flows through 4 states.
3. A person whose parents were citizens of India, 4. Bhima River is an important left-bank tributary of
and the person serves in the Australian Defence the Krishna River.
Forces. Options:
4. A person whose grandparents acquired briefly A. Only 1 statement
(less than 5 years) the citizenship of Pakistan,
before getting the citizenship of Canada. B. Only 2 statements
Options: C. Only 3 statements
A. 1 and 2 D. All 4 statements
B. 1, 2 and 3
C. 1, 2, 3 and 4 97. Which of the following statements is/are true? (Level
– Moderate)
D. 2 and 4
1. CARA (Central Adoption Resource Authority) is a
statutory body established under the Ministry of
93. Which of the following statements is/are true? (Level Women and Child Development.
– Moderate)
2. CARINGS (Child Adoption Resource Information
1. The International Day of Biological Diversity is and Guidance System) is a database of prospective
celebrated on 22nd May every year. children and parents maintained by CARA.
2. The theme for this year was “From Agreement to 3. The Juvenile Justice Act streamlines all adoption
Action: Build Back Biodiversity”. and child safety-related rules in the country.
3. India has a dedicated act for Biodiversity, passed in Options:
the year 2002. A. 1 and 2
Options:
B. 2 and 3
A. 1 and 2 C. 1 and 3
B. 2 and 3 D. 1, 2 and 3
C. 1 and 3
D. 1, 2 and 3 98. How many of the following statements is/are true?
(Level – Easy)
94. The General Data Protection Regulations, in the news 1. Article 123 of the Indian Constitution gives
recently, are related to: (Level – Easy) the President the power for promulgation of
A. Indian government’s proposed regulations for Ordinances.
e-commerce firms. 2. Article 213 gives similar power to a state Governor.
B. WTO’s regulations for cross-border data exchange. 3. The Ordinances must be presented before the
C. Regulations in place by the European Union to Parliament/State legislatures within 6 months of
protect the data of its citizens. their promulgation.
D. A proposed bill in the USA’s Senate regarding net Options:
neutrality. A. Only 1 statement
B. Only 2 statements
95. Consider the following statements: (Level – Moderate) C. All 3 statements
[PYQ 2017]
D. None of the statements
1. In India, the Himalayas are spread over five states
only.
99. Identify the correct statements: (Level – difficult)
2. Western Ghats are spread over five states only.
1. The National Population Register was first prepared
3. Pulicat Lake is spread over two States only. in 1971.
Which of the statements given above is/are correct?
2. It was prepared under the Citizenship Act, of 1955.
A. 1 and 2 only 3. It prepares a database of the citizens of the country.
B. 3 only Options:
C. 2 and 3 only A. 1, 2 and 3
D. 1 and 3 only B. 2 only
C. 2 and 3 only
D. 1 and 3 only
MAY - 2023 14
100. With reference to Swadeshi Movement consider the Options:
following statements: (Level – Easy) [PYQ 2019] A. Only 1 statement
1. It contributed to the revival of the indigenous B. Only 2 statements
artisan crafts and industries.
C. Only 3 statements
2. The National Council of Education was established D. None of the statements
as a part of the Swadeshi Movement.
Which of the statements given above is/are correct?
105. Which of the following brings out the Consumer Price
A. 1 only Index Number for Industrial Workers? (Level – Easy)
B. 2 only [PYQ - 2015]
C. Both 1 and 2 A. The Reserve Bank of India
D. Neither 1 nor 2 B. The Department of Economic Affairs
C. The Labour Bureau
101. Which of the following statements are true? (Level – D. The Department of Personnel and Training
Moderate)
1. 5G technology has lower latency and bandwidth 106. The Project Shivalik, in the news recently, is related
compared to 4G technology. to? (Level – Difficult)
2. India has seen the fastest rollout of the 5G network A. Development of indigenous cryogenic engines in
in the world. India
Options:
B. Deployment of micro-seismographs in the
A. 1 only earthquake-prone Shivalik hills
B. 2 only C. Task force of BRO for construction of roads in
C. Both 1 and 2 Uttarakhand
D. Neither 1 nor 2 D. Conservation of the fragile ecosystems of the
Shivalik range
102. Which of the following statements are true? (Level –
Easy) 107. How many of the following statements is/are true
1. Article 371-C provides for some special provisions about the aircraft carriers of India? (Level – Difficult)
for the state of Manipur. 1. INS Vikrant was the first aircraft carrier of India.
2. These provisions revolve around the development 2. The first indigenously built aircraft carrier is also
of the hill areas of the state. called INS Vikrant.
3. Manipur Hills belong to the Lesser Himalayas. 3. Admiral Gorshkov of Russia was refurbished into
Options: INS Vikramaditya.
A. 1 and 2 4. INS Viraat was originally a part of the British Royal
B. 2 and 3 Navy before it was acquired by India.
Options:
C. 1 and 3
D. 1, 2 and 3 A. Only 1 statement
B. Only 2 statements
103. How many of the following statements is/are true? C. Only 3 statements
(Level – Difficult) D. All 4 statements
1. The Central Board of Film Certification is a statutory
body under the Ministry of Culture. 108. Identify the correct statements: (Level – Moderate)
2. A film can be publicly broadcasted only after the 1. XpoSAT is a mission of ISRO in collaboration with
certification from CBFC. NASA.
3. It provides film certifications in 4 categories. 2. It aims to study bright astronomical X-Ray sources
Options: in space.
A. Only 1 statement Options:
B. Only 2 statements A. 1 only
C. Only 3 statements B. 2 only
D. None of the statements C. Both 1 and 2
D. Neither 1 nor 2
104. How many of the following statements is/are true?
(Level – Easy)
1. The first World Conference on Disaster Risk
Reduction was held in Yokohama.
2. The Sendai Framework emphasized on “Build
Back Better”.
3. The Hyogo Framework on disaster risk reduction
has been adopted by the G7 countries in the latest
summit in Japan.
MAY - 2023 15
109. How many pairs of “Places in News: Country” are 113. Consider the following statements with respect to
correct? (Level – Moderate) Union Public Service Commission (UPSC): (Level-
1. Bakhmut: Russia Easy)
2. Gaziantep: Turkey 1. The UPSC comprises of a chairman and ten
members. Each member holds office for a tenure
3. Nusantara: Indonesia
of 5 years or till he becomes the age of 65 years.
Options:
2. While making reservations of appointments or
A. Only 1 pair posts in favour of any backward class of citizens
B. Only 2 pairs UPSC is consulted.
C. All 3 pairs 3. The Chairman and members of UPSC are not
D. None of the pairs eligible for further employment in the Government
of India or a State.
110. With reference to the cultural history of India, which Which of the statements given above is/are correct?
one of the following is the correct description of the A. 1 only
term paramitas? (Level – Difficult) [PYQ - 2020] B. 1 and 3 only
A. The earliest Dharmashastra texts written in C. 2 and 3 only
aphoristic (sutra) style D. None
B. Philosophical schools that did not accept the
authority of Vedas
114. Which of the following statements is/are incorrect
C. Perfections whose attainment led to the with respect to XPoSat? (Level- Difficult)
Bodhisattva path
1. XPoSat will study various dynamics of bright
D. Powerful merchant guilds of early Medieval South astronomical X-ray sources in extreme conditions
India
2. It has been billed as India’s first, and only the
world’s second polarimetry mission that is meant
111. With respect to the Carbon Border Adjustment to study various dynamics of bright astronomical
Mechanism (CBAM), which of the following X-ray sources in extreme conditions.
statements is/are correct? (Level- Moderate) Options:
1. CBAM is a policy tool introduced by the EU to A. 1 only
reduce carbon emissions by imposing a carbon
tax on imported products, ensuring that they are B. 2 only
subject to the same carbon costs as products C. Both
produced within the EU. D. None
2. It is part of the EU’s “Fit for 55 in 2030 package” to
reduce greenhouse gas emissions by at least 55% 115. With reference to micro-irrigation, which of the
by 2030 compared to 1990 levels. following statements is/are correct? (Level- Easy)
Options: 1. Fertilizer/nutrient loss can be reduced.
A. 1 only 2. It is the only means of irrigation in dry land farming.
B. 2 only 3. In some areas of farming, the receding of the
C. Both groundwater table can be checked.
D. None Select the correct answer using the codes given
below:
112. The Strategic Petroleum Reserves (SPR) facilities A. 1 only
are established in which of the following locations? B. 2 and 3 only
(Level- Difficult) C. 1 and 3 only
1. Chennai D. 1, 2 and 3
2. Mangaluru
3. Mumbai
4. Padur
5. Vishakhapatnam
Options:
A. 1, 2 and 3 only
B. 2, 4 and 5 only
C. 1, 3, 4 and 5 only
D. 1, 2, 3, 4 and 5
MAY - 2023 16
116. With reference to International Organization for Which one of the following is correct in respect of the
Standardization (ISO), Consider the following above statements?
statements: (Level – Difficult) A. Both Statement—I and Statement—II are correct
1. The organization develops and publishes and Statement—II is the correct explanation for
standardization in all technical and nontechnical Statement—I
fields. B. Both Statement—I and Statement—II are correct
2. It is a specialized agency of the United Nations and Statement—Il is not the correct explanation for
created in 1947 and headquartered in Geneva, Statement—I
Switzerland. C. Statement—I is correct but Statement—II is
3. India is a founding member of the International incorrect
Organization for Standardization (ISO). D. Statement—I is incorrect but Statement—II is
How many of the above statements are correct? correct
A. Only one
B. Only two 120. Which one of the following countries has its own
C. All three satellite Navigation System? (Level – Moderate) PYQ
(2023)
D. None
A. Australia
117. Bureau of Indian Standards (BIS) is member of which B. Canada
of the following International Organizations? (Level C. Israel
– Difficult) D. Japan
1. International Organization for Standardization (ISO)
2. International Electrotechnical Commission (IEC) 121. Consider the following Provisions: (Level – Difficult)
3. Pacific Area Standards Congress (PASC) 1. Allocation of seats in the House of the People
4. South Asian Regional Standards Organization 2. Allocation of equitable sharing of time
(SARSO) 3. Electoral roll for every constituency
5. World Standards Service Network (WSSN) 4. Free supply of copies of electoral rolls
Options: How many of these provisions are covered under the
A. 1, 2 and 3 only Representation of the People Act, 1950?
B. 3, 4 and 5 only A. Only one
C. 1, 2, 4 and 5 only B. Only two
D. 1, 2, 3, 4 and 5 C. Only three
D. All four
118. Consider the following statements: (Level – Difficult)
1. Sagar Parikrama is an initiative taken by 122. Consider the following statements with respect to
Government of India, with an aim to resolve the the Official Secrets Act of 1923? (Level – Moderate)
issues of the fishers, other stakeholders and 1. The act applies to servants of the Government and
facilitate their economic upliftment through various to citizens of India outside India
fisheries schemes and programs 2. Lahore Conspiracy Case trial was held by a Special
2. First Phase of “Sagar Parikrama” started from tribunal constituted under the Official Secrets Act.
Sasson Dock, Mumbai. 3. Second Administrative Reforms Commission
3. India is the 3rd largest fish producing country recommended that the Official Secrets Act should
with about 8% share in global fish production and be scrapped.
stands 2nd in aquaculture production. How many of the above statements are incorrect?
How many of the above statements are correct? A. Only one
A. Only one B. Only two
B. Only two C. All three
C. All three D. None
D. None
123. Consider the following statements: (Level – Easy)
119. Consider the following statements: (Level – Moderate)
Statement—I
Statement—I
POCSO Act sets a gender-neutral tone for the legal
The total number of ministers, including the chief
framework available to child sexual abuse victims.
minister, in the council of ministers in a state shall
not exceed 15 per cent of the total strength of the Statement— II
legislative assembly of that state. But, the number of The act does not distinguish between perpetrators of
ministers, including the chief minister, in a state shall child sexual abuse on the grounds of gender, and there
not be less than 15. have been instances where the courts have convicted
Statement—II women for such abuse.
This provision was added by the 91st Amendment Act
of 2003.
MAY - 2023 17
Which one of the following is correct in respect of the Options:
above statements? A. Only 2
A. Both Statement—I and Statement—II are correct B. Only 3
and Statement—II is the correct explanation for
C. Only 4
Statement—I
D. All 5
B. Both Statement—I and Statement—II are correct
and Statement—II is not the correct explanation for
Statement-I 128. Which of the following statements are true? (Level –
C. Statement—I is correct but Statement—II is Difficult)
incorrect 1. Flavanols are a type of nutrient found in some fruits
D. Statement—I is incorrect but Statement—II is and vegetables.
correct 2. They have anti-oxidant properties.
3. Their deficiency in the human body is associated
124. With respect to Comptroller and Auditor General with old age-related memory loss.
(CAG) of India, which of the following statements is/ Options:
are correct? (Level – Easy) A. Only 1 statement
1. The administrative expenses of the office of B. Only 2 statements
the CAG, including all salaries, allowances and C. All 3 statements
pensions of persons serving in that office are
D. None of the statements
charged upon the Contingency Fund of India.
2. The CAG submits three audit reports directly to
the Parliament—audit report on appropriation 129. “Polygenic Risk Score” is: (Level – Moderate)
accounts, audit report on finance accounts, and A. A score given to the countries based on the
audit report on public undertakings. susceptibility of its citizens to get CVDs.
Options: B. An index to determine a region’s vulnerability
A. 1 only towards endogenic disasters.
B. 2 only C. An index to calculate whether or not an asteroid
will strike the Earth.
C. Both
D. A genetic test to determine a person’s susceptibility
D. None
to get heart disease.

125. Salinization occurs when the irrigation water


130. Under which Schedule of the Constitution of India
accumulated in the soil evaporates, leaving behind
can the transfer of tribal land to private parties for
salts and minerals. What are the effects of salinization
mining be declared null and void? (Level – Easy) [PYQ
on the irrigated land? (Level – Moderate) PYQ (2011)
- 2020]
A. It greatly increases the crop production
A. Third Schedule
B. It makes some soils impermeable
B. Fifth Schedule
C. It raises the water table
C. Ninth Schedule
D. It fills the air spaces in the soil with water
D. Twelfth Schedule

126. How many of the following statements are correct?


(Level – Difficult)
1. Goa became a full state of India in 1961.
2. It was a part of the Bijapur kingdom before being
captured by the Portuguese.
3. Operation Vijay was launched in 1961 to expel the
Portuguese from both Goa and Daman and Diu.
Options:
A. Only 1 statement
B. Only 2 statements
C. Only 3 statements
D. None of the statements

127. How many of the following water bodies border


Turkey? (Level – Moderate)
1. Aegean Sea
2. Caspian Sea
3. Bosporus Strait
4. Mediterranean Sea
5. Red Sea
MAY - 2023 18
MAY - 2023 19
SEPTEMBER - 2022
24
MAY - 2023 20

ANSWER KEYS
1. D 41. A
2. A 42. B
3. A 43. A
4. B 44. B
5. B 45. C
6. B 46. B
7. B 47. D
8. C 48. D
9. C 49. C
10. B
50. C
11. D
51. C
12. B
52. D
13. A
53. B
14. B
54. D
15. A
55. C
16. B
56. C
17. A
57. D
18. A
58. C
19. B
59. B
20. C
60. D
21. D
61. C
22. B
62. A
23. D
63. D
24. B
64. B
25. A
65. B
26. B
66. B
27. A
67. C
28. B
68. C
29. B
69. D
30. C
70. B
31. B
71. A
32. D
72. C
33. C
73. B
34. C
74. D
35. D
75. B
36. A
76. C
37. C
77. C
38. C
78. D
39. D
79. D
40. B
80. A
MAY - 2023 21

81. A 123. A
82. B 124. D
83. C 125. B
84. D 126. B
85. A 127. B
86. C 128. C
87. B 129. D
88. C 130. B
89. A
90. C
91. C
92. A
93. D
94. C
95. B
96. D
97. D
98. B
99. B
100. C
101. B
102. A
103. B
104. B
105. C
106. C
107 D
108. B
109. B
110. C
111. C
112. B
113. D
114. D
115. C
116. A
117. D
118. B
119. D
120. D
121. B
122. A
MAY - 2023 22

EXPLANATION FOR THE PRELIMS QUESTIONS


1. Explanation: in most Hindu temples, the image of
● 10 years after the National Food Security Act (NFSA) Jagannath is made of wood and is
was enacted, the union government has revised ceremoniously replaced every twelve or
the nutritional standards of meals at schools and 19 years by an exact replica.
anganwadis. ● Statement 03 is incorrect, The temple is believed
● The revised nutritional standards have augmented to have been constructed in the 12th century by
the proportion of calories and protein, while also King Anatavarman Chodaganga Deva of the
mandating the inclusion of micronutrients in them. Eastern Ganga Dynasty.
● The amendment has been done on the
recommendations of an inter-ministerial committee. 4. Explanation: The Telecom Regulatory Authority of
● The guidelines for nutritional requirements in India (TRAI) was established on 20th February, 1997 by
certain groups, like lower primary and upper the Telecom Regulatory Authority of India Act, 1997.
primary classes, have been updated. Previously,
students in lower primary classes were provided 5. Explanation:
with 450 kilocalories (kcal) and 12 grams of protein ● Mosul & Kirkuk are both in Iraq. Been in news due
for their midday meals. to ISIS controlled territory.
● Now, the protein quantity has been increased to
15-20 gms, while fat (18-21 gms) and carbohydrates 6. Explanation:
(70 gms) are part of the mix as well.
● Statement 1 is not correct, The Government of
● Standards for micronutrients have also been fixed: India launched Mission Karmayogi in September
calcium 170 mg, zinc 2 mg; iron 3.5 mg; dietary 2020, which is a National Programme for Civil
folate 50 micrograms, Vitamin A 100 micrograms; Services Capacity Building.
Vitamin B6 0.43 micrograms; Vitamin B12 0.66
micrograms. ○ Mission Karmayogi is a scheme that exhorts
civil servants to maintain a very high standard
of conduct and behaviour.
2. Explanation:
● Statement 2 is correct, The mission aims to
● Pair 01 is incorrectly matched, Operation Ganga establish a new national architecture for civil
was an evacuation mission carried out by the services capacity building at individual, institutional
Indian government to rescue its citizens stranded and process levels.
in neighboring countries during the 2022 Russian
● Statement 3 is correct, iGOT Karmashala is a
invasion of Ukraine.
consultative workshop conducted as a part of the
● Pair 02 is correctly matched, Operation Kaveri is mission.
a codename for India’s evacuation effort to bring
back its citizens stranded in Sudan amid intense 7. Explanation:
fighting between the army and a rival paramilitary ● Statement 1 is not correct, A patent is an exclusive
force in April 2023. right granted for an invention which includes a
● Pair 03 is incorrectly matched, Operation Devi product or a process that generally provides a new
Shakti was an operation of the Indian Armed Forces way of doing something, or offers a new technical
to evacuate Indian citizens and foreign nationals solution to a problem.
from Afghanistan after the collapse of the Islamic ○ Discoveries however are not patentable and
Republic of Afghanistan and the fall of Kabul to the are excluded from patent protection.
Taliban.
● Statement 2 is correct, The term for protection for
a product patent according to Indian Patent Act
3. Explanation: 2005 is 20 years.
● Statement 01 is correct, The Jagannath Temple is
an important Hindu temple dedicated to Jagannath, 8. Explanation:
a form of Vishnu. It is called ‘Yamanika Tirtha’ ● Statement 1 is correct, Malthus stated that
where, according to the Hindu beliefs, the power population increased in a geometric progression
of ‘Yama’, the god of death, has been nullified in i.e. 2, 4, 16, 132, while resources such as food
Puri due to the presence of Lord Jagannath. production increased in arithmetic progression i.e.
● Statement 02 is correct, This temple was 2, 4, 6, 8.
called the “White Pagoda” and is a part of Char ○ Thus according to Malthusian Population
Dham pilgrimages (Badrinath, Dwaraka, Puri, theory, the population grew faster than food
Rameswaram). production and tended to outstrip it in a short
○ The Puri temple is famous for its annual Ratha time.
Yatra, or chariot festival, in which the three ● Statement 2 is not correct, The Total Fertility Rate
principal deities are pulled on huge and is the average number of children that a woman
elaborately decorated temple cars. will have during her lifetime.
■ Unlike the stone and metal icons found ○ It is measured in terms of children per woman.
MAY - 2023 23
● Statement 3 is correct, According to the United 12. Explanation:
Nations Population Division, a Total Fertility Rate of ● Statement 1 is correct, The Supreme Court in a
about 2.1 children per woman is called replacement- landmark judgement in the Vishakha and others
level fertility. v/s State of Rajasthan 1997 case came up with
○ Replacement Level Fertility is the level of ‘Vishakha guidelines’.
fertility at which a population exactly replaces ○ These guidelines formed the basis for the
itself from one generation to the next. Sexual Harassment of Women at Workplace
(Prevention, Prohibition and Redressal) Act,
9. Explanation: 2013 (POSH Act).
● Statement 1 is not correct, The Union Ministry ● Statement 2 is correct, According to Section
of Urban Development takes up the Swachh 4 of the POSH Act, 2013, the employer of every
Survekshan in urban areas organisation is required to establish an Internal
○ The Ministry of Drinking Water and Sanitation Complaints Committee (ICC).
takes up the Swachh Sarvekshan in rural areas. ● Statement 3 is not correct, The Internal Complaints
● Statement 2 is not correct, A city/ward can be Committee must constitute the following members:
notified/declared as an ODF city/ODF ward if, at ○ Presiding Officer: One: a woman employed at
any point of the day, not a single person is found a senior level at the workplace from amongst
defecating in the open. employees.
● Statement 3 is correct, A city/ward can be notified/ ○ Members: not less than two: from employees
declared as ODF+ city/war if, at any point of the preferably committed to the cause of women
day, not a single person is found defecating and/ or who have experience in social work or have
or urinating in the open AND all community and legal knowledge.
public toilets are functional and well maintained. ○ Member: One: from amongst non-
governmental organizations or associations
10. Explanation: committed to the cause of women or a person
familiar with the issues relating to sexual
● The substitution of steel for wooden plough in
harassment
agricultural production is an example of Capital-
augmenting technological progress. ○ The Act provides that at least one-half of the
total Members so nominated shall be women.
● Capital-augmenting technological progress
ensures more productive use of existing capital ● Statement 4 is correct, The POSH Act provides a
goods. special redress mechanism for complaints in the
unorganised sector.
● Replacement of wooden ploughs by steel ploughs
increases productivity. ○ Section 7 of the Act mandates the establishment
of a Local Complaints Committee by the State
Government.
11. Explanation:
● The Kisan Credit Scheme (KCC) was introduced
13. Explanation:
in 1998 based on a model scheme prepared
by the National Bank for Agriculture and Rural ● The Yanomami Area is the largest indigenous
Development (NABARD). territory in Brazil.
● The scheme was introduced to meet the various ● It is located in the northern part of the country, on
credit requirements of the agriculture sector by the border of Venezuela.
giving financial assistance to farmers. ● Close to about 20 thousand indigenous people
● Eligibility Criteria are as follows for the Kisan Credit live in this area.
Card Scheme: ● Illegal miners, pollution of rivers and large-scale
○ Individual farmers who are owners/cultivators deforestation have threatened the existence of the
indigenous populations in the area.
○ Sharecroppers, tenant farmers
○ Self-Help Groups of sharecroppers, farmers,
tenant farmers, etc 14. Explanation:
○ Farmers involved in the production of crops or
activities such as animal husbandry
○ Fish farmers, fishers, SHGs, JLGs, and women
groups
○ Fishermen who own a registered boat or any
other type of fishing vessel and possess the Source: www.worldbank.org
necessary license or permissions for fishing in
estuaries or the sea.
○ Poultry farmers and even those who raise
sheep, rabbits, goats, pigs, etc.
○ Dairy: Farmers, dairy farmers, SHGs, JLGs, and
tenant farmers who own, lease, or rent sheds.
MAY - 2023 24
15. Explanation: 18. Explanation:
● Statement 1 is correct, Due to less landmass, the ● Statement 01 is correct, The Aspirational Blocks
winds experience a lesser frictional force in the Programme is on the lines of the Aspirational
Southern Hemisphere compared to the Northern District Programme that was launched in 2018 and
Hemisphere. covers 112 districts across the country.
● Statement 2 is not correct, The Coriolis effect is an ○ It is aimed at improving the performance
apparent force, relative to the earth’s surface, that of blocks lagging on various development
causes deflection of moving objects to the right parameters.
in the Northern Hemisphere and to the left in the ○ This will enable holistic development in those
Southern Hemisphere due to the earth’s rotation. areas that require added assistance.
○ The magnitude and intensity of the Coriolis ○ The program will cover 500 districts across 31
Force are identical in both Hemispheres. states and Union Territories initially.
○ Over half of these blocks are in 6 states—
16. Explanation: Uttar Pradesh (68 blocks), Bihar (61), Madhya
● Statement 1 is incorrect, In a free-market economy, Pradesh (42), Jharkhand (34), Odisha (29) and
the exchange rate is decided by the supply and West Bengal (29).
demand for rupees and dollars. However, in India, ● Statement 2 is incorrect, Maharashtra has
the exchange rate is not fully determined by the conceptualised and initiated the Aspirational Cities
market. From time to time, the RBI intervenes in the Programme (ACP) modelled on the lines of the
foreign exchange (forex) market to ensure that the Aspirational Districts initiative of Niti Aayog.
rupee “price” does not fluctuate too much or that it
doesn’t rise or fall too much all at once.
19. Explanation:
○ Currently, the Central Board of Indirect
Taxes and Customs manually notifies the
rate of exchange of conversion of 22 foreign
currencies every fortnight.
● Statement 2 is correct, Devaluation means
officially lowering the value of the currency in terms
of foreign exchange. The devaluation of currency
is done by the government.
○ The Indian rupee was devalued in the years
1949, 1966, and 1991 by the government of
India.

17. Explanation: Source: World Atlas


● Statement 1 is correct, Type 1 diabetes (T1D) is an
organ-specific autoimmune disease caused by the 20. Explanation: Subsidiary Alliance was basically a treaty
autoimmune response against pancreatic β cells. between the British East India Company and the Indian
○ It is a condition where the pancreas completely princely states, by virtue of which the Indian kingdoms
stops producing insulin, the hormone lost their sovereignty to the English. It also was a major
responsible for controlling the level of glucose process that led to the building of the British Empire in
in the blood by increasing or decreasing India.
absorption to the liver, fat, and other cells of The Subsidiary Alliance system was used by Lord
the body. Wellesley to build an empire in India.
● Statement 2 is incorrect, Type 2 diabetes is
caused by a combination of two primary factors: The Subsidiary Alliance Treaty
defective insulin secretion by pancreatic β-cells ● Acknowledge the British East India Company as
and the inability of insulin-sensitive tissues to the paramount power
respond appropriately to insulin. ● Permanently station a British army within his
○ In type 2 diabetes, the body isn’t able to use territory
insulin the right way. This is called insulin ● Pay subsidy for maintaining this army.
resistance. As type 2 diabetes gets worse, the
pancreas may make less and less insulin. This ● Post a British resident in his court.
is called insulin deficiency. ● Not negotiate with any other rulers without prior
● Statement 3 is incorrect, Type 1 diabetes, accounts consent from the Governor-General.
for 5 to 10 out of 100 people who have diabetes. ● Not employ any European in their service without
getting approval from the British.
● If the ruler fails to make payment as per the
Alliance, then the state would be confiscated by
the British East India Company.
MAY - 2023 25
21. Explanation: spina bifida, and haemophilia are some
● All the above projects are developed by the Border examples of rare diseases.
Roads Organisation. ● Categorization of rare diseases according to the
● Project Beacon was launched in the 1960s. The national policy of rare diseases.
project currently looks after road infrastructure ○ Group 1: Disorders amenable to one-time
development and maintenance in important areas curative treatment.
of Kashmir. ○ Group 2: Those requiring long-term or lifelong
● Project Vijayak along with Project Himank are treatment.
responsible for the construction and maintenance ○ Group 3: Diseases for which definitive treatment
of critical road infrastructure in Ladakh. is available but challenges are to make optimal
● Project Dantak is a joint initiative by the Border patient selection for benefit, very high cost and
Roads Organisation (BRO) of India and the Royal lifelong therapy.
Government of Bhutan for the construction and
maintenance of roads and infrastructure in Bhutan. 24. Explanation:
The project was started in 1961 and has since then
been responsible for the construction of over 1,500 ● Statement 01 is incorrect, The Loyumba Shinyen is
km of roads and 168 bridges in Bhutan. an 11th-12th century written constitution, regulated
in the Manipur kingdom during the regime of King
● Project Sampark was raised by BRO in 1975. It Loyumba (1074 AD-1112 AD).
looks after the construction, improvement and
maintenance of approximately 2,600km of strategic ○ Pitambar Charairongba also known as
roads in the border district of Jammu, Kathua, “Eningthou Ningthem Charairongba” was the
Doda, Udhampur, Rajouri, Reasi and Poonch. Meitei king and the ruler of Kangleipak from
1697 to 1709.
○ Ujh Bridge and Basantar Bridge in Jammu &
Kashmir which was constructed under this ● Statement 02 is correct, China’s trade with Manipur
project were inaugurated in 2019. has brought about certain economic innovations
like the introduction of silk and silkworm and
● Project Vartak was formed in 1960 as Project gunpowder. Manipur learned the art of making
Tusker and later renamed Project Vartak in 1963. gunpowder from the Chinese merchants.
○ It is the first established project of the ● Statement 03 is correct, In the First Anglo-Burmese
Border Roads Organisation. Its task was later War, the British helped Prince Gambhir Singh regain
expanded to construct and maintain roads in his kingdom of Manipur, which had been heretofore
Arunachal Pradesh and adjoining districts of occupied by the Burmese. Subsequently, Manipur
Assam. became a British protectorate.

22. Explanation: Article 355 of the Indian Constitution is 25. Explanation: A geostationary orbit is a circular
a provision that empowers the central government geosynchronous orbit in which the satellite is placed at
to take all necessary measures to protect any state an altitude of approximately 35786 km above mean sea
against external aggression and internal disturbance. level. The orbit lies in the plane of the Earth’s equator.
● This provision gives the central government the An object in such an orbit has an orbital period equal to
authority to intervene in the affairs of a state if Earth’s rotational period and thus appears motionless.
there is any threat to national security or law and
order. 26. Explanation:
● The central government can take various measures, ● Statement 1 is not correct, Space Science and
such as deploying armed forces, to protect a state Technology Awareness Training (START) is an
from external aggression or internal disturbance. introductory-level online training programme
● However, the provision also imposes certain launched by the ISRO.
restrictions on the central government’s powers. ○ The programme is aimed at post-graduate and
The central government can only use its powers final-year undergraduate students of physical
under Article 355 if the state government has sciences and technology and will be delivered
requested its assistance or if the President, on by scientists from Indian academia and ISRO.
receiving a report from the Governor or otherwise, ● Statement 2 is correct, The START programme
is satisfied that the situation in a state warrants is designed in such a way that it covers various
such intervention. aspects of space science, such as Astronomy,
Astrophysics, Aeronomy, Heliophysics, Sun-Earth
23. Explanation: interaction and Instrumentation.
● The World Health Organisation defines a rare
disease as an often debilitating lifelong disease or 27. Explanation:
disorder condition with a prevalence of 1 or less, ● “Shoot-at-sight” orders can be passed under:
per 1000 population. ○ Sections 41-60 and Sections 149-152 of the
● Most rare diseases are genetic in origin and thus Code of Criminal Procedure (CrPC), 1973.
are present throughout the person’s entire life, ○ Section 3(a) of the Armed Forces Special
even if symptoms do not immediately appear. Powers Act, 1958 (AFSPA),
○ Lysosomal Storage Disorders (LSD), Pompe ○ Section 81 of the Indian Penal Code (IPC),
disease, cystic fibrosis, muscular dystrophy, 1860
MAY - 2023 26
28. Explanation: ● India recently won 10 gold medals at Moscow Star
● A 18-foot-high obelisk, made of grey Aberdeen Wushu Championship 2023.
granite and standing on a sandstone base, was
unveiled by Lord Curzon on April 19, 1902 inside 32. Explanation:
the Kashmere Gate Telegraph Office. ● Statement 1 is correct, The I2U2 grouping was
● The words “The Electric Telegraph has saved formed in October 2021.
India” are inscribed on the Telegraph Memorial ● Statement 2 is not correct, I2U2 is an alliance
which is dedicated to the efforts and sacrifice of created by four countries namely – India, Israel,
telegraphers during the Revolt of 1857. the United Arab Emirates (UAE) and the United
States (US)
29. Explanation: ● Statement 3 is correct, The four-nation coalition
● Pair 1 is correct, Chandauli is an aspirational district, is also known as the ‘International Forum for
also known as the “rice bowl of Uttar Pradesh”, the Economic Cooperation’ or the ‘West Asian Quad’.
District Administration of Chandauli through the
“Chandauli Black Rice Initiative” tried to increase 33. Explanation:
the income of the farmers by employing innovative
methods. ● Statement 1 is correct, Shikharas is a common
element in the many styles of Hindu temple
● Pair 2 is correct, Kalika Chetarike was launched by architecture, of which the three most common are
the Karnataka Government in order to bridge the Nagara, Vesara, and Dravidian.
learning gap induced by the Covid pandemic.
○ Nagara Style usually has multiple Shikharas
● Pair 3 is not correct, Santhe Kaushalkar is the whereas the Dravidian style typically has only
first of its kind SHG/Artisan profiling platform and one Shikhara.
mobile application offering SHGs/Artisans a digital
identity to showcase their products, thus increasing ● Statement 2 is correct, In the Dravidian Style of
their visibility and value. temple architecture, the most prominent element
is the Gopuram.
○ With this application, Government of Karnataka
and UNDP are providing business connections
to match directly with the customers and 34. Explanation:
compete effectively in the online market. ● Israel is not a member of the Arab League.
● Pair 4 is not correct, SARTHI & SAKHI -Mental
Health Helpline are dedicated mental health
helplines established for individuals to access
mental health care from a professional in Covid-19
times.
○ Nodal agency is the District administration,
Ghaziabad, Uttar Pradesh.

30. Explanation:
● Statement 1 is correct, According to Article 30 of
the Constitutions, “All minorities shall have the right
to establish and administer educational institutions
Source: National Geographic
of their choice”.
● Statement 2 is not correct, There is no provision 35. Explanation:
for the President of India to automatically nominate ● The Standards & Labelling Programme is one of
a member of a minority religious community to the the major thrust areas of the Bureau of Energy
Lok Sabha. Efficiency.
● Statement 3 is correct, Minorities can derive ● A key objective of the Star Labelling Programme is
benefits from the Prime Minister’s 15-Point to provide the consumer with an informed choice
Programme which was launched in 2005. about energy saving and thereby the cost-saving
○ The programme envisages to ensure the potential of the relevant marketed product.
welfare of minorities in the fields of education, ● The appliances that require mandatory energy
skill development, employment and prevention labelling include:
of communal conflicts.
○ Room Air Conditioners
○ Frost Free Refrigerators
31. Explanation:
○ Tubular Fluorescent Lamp
● Wushu is a collection of Chinese martial arts.
○ Distribution Transformer
● Wushu in Chinese literally means “martial
technique”. ○ Direct Cool Refrigerator
● It has a history dating back thousands of years to the ○ Colour TV
Qin Dynasty. The International Wushu Federation ○ Electric Geysers
was formed in 1990 to govern the distinct types of ○ LED Lamps
martial arts under the “wushu” banner. ○ Ceiling Fans
MAY - 2023 27
36. Explanation: 40. Explanation:
● Statement 1 is correct, Prime Minister of India ● Statement 1 is not correct, The New Development
announced Mission LiFE at the UN Climate Change Bank (NDB) is a multilateral financial institution
Conference (UNFCCC) – COP26 held in Glasgow, established by the BRICS countries, namely, Brazil,
United Kingdom, 2021. Russia, India, China and South Africa.
● Statement 2 is correct, Mission LiFE intends to ● Statement 2 is correct, NDB is headquartered in
democratise the battle against climate change and Shanghai, China.
achieve sustainable development goals (SDGs).
● Statement 3 is not correct, Mission LiFE does not 41. Explanation:
focus on biodiversity conservation directly. ● Statement 1 is correct, The Indian LIGO
comprises two 4-km-long vacuum chambers, built
37. Explanation: perpendicular to each other that constitute the
● Statement 1 is correct, India has bi-annual or dual most sensitive interferometers in the world.
cyclone seasons namely – pre-monsoon (April- ● Statement 2 is correct, LIGO-India will be located
June) and post-monsoon. in the Hingoli district of Maharashtra, about 450
● Statement 2 is correct, the Bay of Bengal km east of Mumbai.
experiences more frequent cyclones as compared ○ The government has recently approved the
to the Arabian Sea on account of high sea surface construction of the project and it will be built
temperature. by the Department of Atomic Energy and the
● Statement 3 is correct, Cyclone Mocha will be the Department of Science and Technology with
first cyclonic storm of the year 2023. the U.S. National Science Foundation and
several national and international research
○ The name “Mocha” (pronounced as Mokha)
institutions.
was suggested by Yemen.
● Statement 3 is not correct, The LIGO in the US first
detected gravitational waves in 2015, which led to
38. Explanation: a Nobel Prize in Physics in 2017.
● Statement 1 is correct, Rabindranath Tagore, in
1913, became the first non-European to receive
42. Explanation:
a Nobel Prize in Literature and the second non-
European to receive a Nobel Prize after Theodore ● Statement 1 is correct, Section 144 of the CrPC
Roosevelt. empowers a district magistrate, a sub-divisional
magistrate, or any other executive magistrate
○ He was awarded the Nobel Prize in Literature
empowered by the state government, to issue
for “Gitanjali”, a collection of poetry.
orders to prevent and address urgent cases of
● Statement 2 is not correct, Rabindranath Tagore apprehended danger or nuisance.
was given a knighthood for services to Literature
in 1915 by King John V and became the first Indian
○ This law empowers the magistrate of a State or
UT to pass an order prohibiting the gathering
to receive the knighthood.
of four or more people in a specified area.
○ He, however, returned his knighthood in 1919
because of the Jallianwala Bagh massacre in
● Statement 2 is not correct, As per the order under
the said section, there shall be no movement of
protest of the mass murder of the people.
public and all educational institutions shall also
● Statement 3 is correct, Manasi is a collection of remain closed.
poems authored by Rabindranath Tagore.
● Statement 3 is not correct, The Section also
empowers authorities to block internet access in
39. Explanation: the region.
The statutory functions of the Enforcement Directorate
(ED) include the enforcement of the following Acts: 43. Explanation:
● The Foreign Exchange Regulation Act, 1973 (FERA) ● In diplomacy, a “persona non grata” refers to a
● The Foreign Exchange Management Act, 1999 foreign diplomat who is asked to be recalled to
(FEMA) their home country by their host country.
● The Prevention of Money Laundering Act, 2002 ● If the person is not recalled as requested, the
(PMLA) host state may refuse to recognize the person
● The Fugitive Economic Offenders Act, 2018 concerned as a member of the diplomatic mission.
(FEOA)
● The ED is also a sponsoring agency under the 44. Explanation:
Conservation of Foreign Exchange and Prevention ● National Company Law Tribunal (NCLT) is a quasi-
of Smuggling Activities Act, 1974 (COFEPOSA). judicial authority incorporated for dealing with
corporate disputes that are of civil nature arising
under the Companies Act.
● A person aggrieved by an order of the National
Company Law Appellate Tribunal (NCALT) may file
an appeal to the Supreme Court.
MAY - 2023 28
● NCLAT is the Appellate Tribunal for hearing 48. Explanation:
appeals against the orders passed by NCLT(s) ● Statement 1 is correct, Reduction in or waiving
under Section 61 of the Insolvency and Bankruptcy of import duties helps arrest high inflation in the
Code, 2016 (IBC). domestic market.
● NCLAT is also the Appellate Tribunal for hearing ● Statement 2 is correct, Since the waiving of import
appeals against the orders passed by the duties reduces the prices of the products, the
Insolvency and Bankruptcy Board of India (IBBI) spending capacity of the citizens increases.
under Section 202 and Section 211 of IBC. ● Statement 3 is correct, An import duty has a
● NCLAT was established under Section 410 of the perverse impact on the balance of payments hence
Companies Act, 2013 for hearing appeals against waning import duties can disturb the country’s
the orders of the National Company Law Tribunal(s) balance of payment.
(NCLT), with effect from 1st June 2016. ● Statement 4 is correct, Freezing import duties
and reducing duty slabs can provide a boost to
45. Explanation: domestic manufacturing industries.
● Statement 1 is correct, All plant cells consist of a
hard layer outside the cell membrane called a cell 49. Explanation:
wall. Animal cells lack a cell wall. ● Statement 1 is correct, The Cham dance is a form
● Statement 2 is not correct, Plasma membranes of dance that is performed by Tibetan Buddhists
are present both in plant as well as animal cells. during Buddhist festivals, such as Losar (the
● Statement 3 is correct, In plants, a single, large Tibetan New Year), and Buddha Jayanti.
and permanent vacuole is present. ● Statement 2 is not correct, The Cham Lama dance
○ In animal cells, vacuoles, if present are small is popular in Tibetan settlement areas of Himachal
and many.​ Pradesh, such as Lahaul & Spiti, Ladakh and
Kinnaur.
46. Explanation: ● Statement 3 is correct, The performance involves
● Statement 1 is not correct, The term “Biodiversity monks in elaborate and colourful masks, headgear
hotspots” was coined by a British environmentalist and costumes dancing slowly to the music played
named Norman Myers. using traditional instruments such as drums and
cymbals also performed by monks.
○ “Biodiversity hotspots” are defined as regions
which are known for their high species richness
and endemism. 50. Explanation:
● Statement 2 is correct, Around the world, there ● The Pole Star’s position with respect to the rotating
are currently 36 biodiversity hotspots that account earth never changes and it always points in the
for 2.5% of Earth’s land surface. north direction and the Pole Star is in the rotation
axis of the sky.
● Statement 3 is correct, There are four biodiversity
hotspots in India namely – the Himalayas, Indo- ● In ancient times, people used the position of the
Burma Region, the Western Ghats and the Pole star to determine directions during the night.
Sundaland. ● Since the Pole star always points towards the
northern direction, walking in the direction keeping
47. Explanation: the pole star to his left will help the person reach
his village located in the east.
● Pair 1 is correct, The River Rio Grande Marks the
boundary between Mexico and the United States.
51. Explanation:
● Pair 2 is correct, The Orange River is a river in
Southern Africa that forms part of the international The Fisher Effect is an economic theory created by
borders between South Africa and Namibia. economist Irving Fisher that describes the relationship
between inflation and both real and nominal interest
○ It also forms a boundary between South Africa
rates. The Fisher Effect states that the real interest rate
and Lesotho.
equals the nominal interest rate minus the expected
● Pair 3 is correct, The Amur is the world’s tenth- inflation rate.
longest river that forms a border between the
Russian Far East and Northeastern China.
52. Explanation:
Section 167 CrPC says that an arrested person will be
entitled to default bail after 90 days in cases where
the investigation relates to an offence punishable
with death, imprisonment for life or imprisonment for a
term of not less than 10 years and 60 days, where the
investigation relates to any other offence.
● It is granted to both bailable and non-bailable
offences.

Source: Geographyrealm.com
MAY - 2023 29
53. Explanation: 59. Explanation:
● Every employer must constitute an Internal ● Saturn, which is the sixth planet from the sun in
Complaints Committee (ICC) at each office or our solar system, is often called “The Jewel of the
branch that has 10 or more employees. Solar System” because of its beautiful rings.
Read more on the Prevention of Sexual Harassment ● Saturn is the second largest planet in our solar
(PoSH) Act, 2013. system.
● Saturn is named after the Roman god of agriculture.
54. Explanation: ● Saturn has the lowest density of all the planets in
The United Kingdom financial regulator that oversees our solar system. Its density is lesser than water.
LIBOR has announced that they’ll discontinue the index
by June 30, 2023. 60. Explanation:
● Statement 1 is not correct, Ionosphere facilitates
55. Explanation: radio communication because it reflects and
Microbial fuel cells are bio-electrochemical systems modifies radio waves used for communication and
that drive a current by using bacteria and taking away navigation.
bacterial interactions from nature. These devices ○ The Ozone layer is found in the Stratosphere.
consist of anode and cathode compartments separated
● Statement 2 is not correct, Radio waves are a
by cation specific membrane which converts chemical
type of electromagnetic radiation with the longest
energy into electrical energy through a catalytic reaction
wavelengths in the electromagnetic spectrum.
of microorganisms, which makes statement 1 correct.
The substrates used in such cells are organic in nature ○ However, it is not the reason why the
such as glucose, acetate, arabitol, carboxymethyl ionosphere facilitates radio communication.
cellulose, corn stover biomass, etc. which makes
statement 2 incorrect. Statement 3 is correct as such 61. Explanation:
cells are used in the treatment of wastewater. ● The UN Framework Convention on Climate Change
(UNFCCC) defines ‘Just Transition’ as the move
56. Explanation: towards an environmentally sustainable economy
● Statement 1 is correct, Khooni Darwaza, which is while contributing to the goals of decent work for
located near the Delhi Gate, was built during the all, social inclusion and the eradication of poverty.
reign of Sher Shah Suri in the 16th century. ● A Just Transition refers to the greening of an
● Statement 2 is not correct, The architecture of economy in a way that is as fair and inclusive as
Khooni Darwaza is a blend of Mughal and Afghan possible to everyone concerned, creating decent
styles. work opportunities and leaving no one behind.
● Statement 3 is correct, Khooni Darwaza gets its
name from the killings that took place there during 62. Explanation:
the Indian Rebellion of 1857. ● Statement 1 is correct, As per the Model Code
of Conduct, the Ministers shall not combine their
57. Explanation: official visit with electioneering work and shall not
also make use of official machinery or personnel
● As per the Conduct of Elections Rules, 1961, the
during the electioneering work.
persons are who are entitled to vote by post
include: ● Statement 2 is correct, The party or candidate
should inform the local police authorities of the
○ Special voters;
venue and time of any proposed meeting well in
○ Service voters; time so as to enable the police to make necessary
○ Voters on election duty; and arrangements for controlling traffic and maintaining
○ Electors subjected to preventive detention peace and order.
● Statement 3 is not correct, A political party
58. Explanation: member urging the voters to go and cast their
vote in maximum numbers is not considered as a
● There are a total of nine nations that possess violation of the Model Code of Conduct.
nuclear weapons.
● The five nuclear-weapon states (NWS) officially
recognized as possessing nuclear weapons by the 63. Explanation:
Nuclear Non-Proliferation Treaty (NPT) are: ● Statement 1 is not correct, WPI captures the
○ China average movement of wholesale prices of goods
for bulk sale at the level of the early stage of
○ France transactions.
○ Russia ○ Thus it captures changes in the prices of
○ The United Kingdom goods and services after they enter the retail
○ The United States market.
● The remaining four nations namely – India, ● Statement 2 is not correct, The Reserve Bank
Pakistan, Israel and North Korea have not signed of India (RBI) currently uses the Consumer Price
the treaty and thus are not a party to the treaty. Index (CPI) for inflation targeting.
MAY - 2023 30
64. Explanation: 67. Explanation:
● Statement 1 is correct, Mahadayi or Mandovi or ● Pair 1 is not correct, Cherry Blossom Showers are
Mhadei river originates in the Bhimgad Wildlife witnessed in Kerala and some parts of Karnataka.
Sanctuary located in the Belagavi district of ● Pair 2 is correct, Bordoisila is the Assamese word
Karnataka. for pre-monsoon showers.
● Statement 2 is not correct, Mahadayi is a west- ● Pair 3 is correct, Mango Showers are pre-
flowing river that enters Goa from Sattari taluk of monsoon showers which are common in the states
the North Goa district and it finally joins the Arabian of Karnataka and Kerala.
Sea at Panaji.
● Pair 4 is correct, The summer pre-monsoon
○ The Mandovi river basin falls in the states of showers in the coffee-growing regions of Karnataka
Goa, Karnataka and Maharashtra. are called Coffee Showers.
● Statement 3 is correct, Salim Ali Bird Sanctuary is
located on the western tip of the Island of Chorão 68. Explanation:
along the Mandovi River in Goa.
● Statement 1 is not correct, India’s first nuclear test
conducted in 1974 was codenamed “Operation
65. Explanation: Smiling Buddha”.
● Statement 1 is not correct, The DMH-11 (GM ○ Pokhran-II was the series of five nuclear bomb
mustard) is produced as a result of a crossing test explosions that were conducted by India
between two varieties of mustard namely the in May 1998 at the Indian Army’s Pokhran
Varuna and Early Heera-2. Test Range, under the assigned code name
○ To facilitate such crossing, genes from two soil Operation Shakti.
bacterium called Barnase and Barstar were ● Statement 2 is not correct, The bomb was
introduced. detonated in the army base Pokhran Test Range,
○ Barnase in Varuna induces temporary sterility in Rajasthan, by the Indian Army under the
and restricts its natural self-pollination traits supervision of several key Indian generals.
and tendencies. ● Statement 3 is correct, India is not a signatory to
○ On the other hand, Barstar limits the effects of the Nuclear Non-Proliferation Treaty (NPT).
Barnase on Heera thereby allowing seeds to
be produced. 69. Explanation:
● Statement 2 is correct, GM mustard has the ● Statement 1 is correct, Endosulfan is an
genes that allow the plant cross-pollination and organochlorine insecticide and acaricide that
hybridization. is being phased out globally due to its acute
● Statement 3 is not correct, GM mustard has neurotoxicity.
been developed by researchers at the Centre ● Statement 2 is correct, Endosulfan is listed under
for Genetic Manipulation of Crop Plants at the both Stockholm Convention on Persistent Organic
University of Delhi. Pollutants as well as the Rotterdam Convention on
Prior Informed Consent.
66. Explanation: ● Statement 3 is correct, In India, a 2011 Supreme
● Statement 1 is not correct, Sodium-ion batteries Court order has banned the production, sale and
are a promising alternative to lithium-ion batteries, use of Endosulfan.
which is currently the most widely used type of
rechargeable battery. 70. Explanation:
○ Sodium is more than 500 times more ● The Congress session was held at Lahore in 1929
abundant than lithium, which is available in a and Pandit Nehru was the president of the session.
few countries.
● This session was very significant because in
● Statement 2 is correct, India has an abundance of this Lahore session, the prominent party Indian
Na sources, which renders the upcoming Na-ion National Congress took the resolution of Poorna
battery system extremely important in the Indian Swaraj or complete independence.
context.
● The Congress approved a motion for complete
○ The Geological Survey of India (GSI) recently independence and also the President of Congress
announced that they had ‘found’ minerals hoisted the flag of ‘Complete Freedom’ on the
bearing Lithium deposits in the Reasi district of bank of the Ravi at midnight of 31 December 1929,
Jammu. in front of huge crowds.
○ But, Sodium is much more easily available in
India.
● Statement 3 is correct, India imports about 70%
of its Li-ion cell requirement from China and Hong
Kong.
○ India is said to be one of the largest importers
of Lithium batteries in the world.
MAY - 2023 31
71. Explanation: ● Their thick white-grey coat spotted with large
● Statement 1 is correct, According to WHO, black rosettes not just protects them from the
Cardiovascular diseases (CVDs) are the leading cold but also makes them almost invisible in their
cause of death globally, taking an estimated 17.9 surroundings providing natural camouflage. Thus,
million lives each year. Snow Leopards are often termed “ghosts of the
● Statement 2 is correct, India Hypertension mountains”.
Management Initiative (IHMI) was launched with ● Conservation statue:
the aim to reduce disability and death related to ○ IUCN Red List Status: Vulnerable
cardiovascular disease (CVD). ○ WPA, 1972 Schedule: Schedule I Protection
● Statement 3 is not correct, To mark World ○ CITES: Appendix 1 Protection
Hypertension Day, the Union Health Ministry
launched an ambitious “75/25” initiative for
screening and putting 75 million people with 75. Explanation:
hypertension and diabetes on Standard Care by Assets: Assets are such items possessed by the
2025. banks that will provide benefits in future.
● Assets of a bank include Advances, Investments,
72. Explanation: Loans lent, Money at call and short notice, etc.
● Statement 1 is not correct, Greater Flamingos are
endemic to the Afro-Eurasia region. Liabilities: Liabilities are items that are obligations
○ Greater flamingos migrate to India in search of for a bank. In simple terms, liabilities are what the bank
food and to lay eggs. owes to others.
● Statement 2 is not correct, Out of six species ● Deposits are amounts to be payable to the
of Flamingos found across the world, two can depositor i.e. the bank has to repay the amount to
be spotted in India which include the Greater the depositor on demand or on the maturity of the
Flamingos and the Lesser Flamingos. deposit. Thus, deposits are a liability for the bank.
● Statement 3 is not correct, Flamingo populations
are mainly found in northwestern parts of India. 76. Explanation:
○ Flamingos in India can be seen in: Thol Lake, ● Statement 1 is correct, With respect to the duration
Nal Sarovar and Khijadiya Bird Sanctuary of treatment, MAM may require only short-term
(Gujarat), Thane Creek and Sewri Mudflats in treatment while the duration of treatment for SAM
Mumbai (Maharashtra), Najafgarh Lake (Delhi), is usually longer than for MAM.
Chilika Lake (Odisha) and Pulicat Lake (Andhra ● Statement 2 is correct, MAM can often be managed
Pradesh) with simpler interventions, such as supplemental
● Statement 4 is correct, The Greater Flamingo is feeding, nutrient-rich foods, and education on
listed as “Least Concerned” under the IUCN Red optimal infant and young child feeding practices.
list. ○ However, SAM requires specialized medical
care, such as inpatient or outpatient therapeutic
73. Explanation: feeding, antibiotics, and other supportive care.
● Statement 1 is not correct, The Nutrient 77. Explanation:
Based Subsidy scheme is administered by the
● Statement 1 is correct, Pir Panjal is the largest
Department of Fertilisers under the Ministry of
range of the Lesser Himalayas.
Chemicals & Fertilisers.
● Statement 2 is not correct, The Pir Panjal Pass
● Statement 2 is correct, Under the scheme,
helps connect the Kashmir Valley with Rajouri and
fertilisers are provided at subsidised rates based
Poonch via the Mughal Road.
on the nutrients contained, namely Nitrogen (N),
Phosphate (P), Potash (K) and Sulphur (S). ● Statement 3 is correct, The renowned Galyat
mountains are also located in the Pir Panjal Range.
○ The Scheme does not include urea-based
fertilisers.
78. Explanation:
74. Explanation: ● Statement 1 is correct, Political parties are
provided free airtime on public broadcasters such
● Snow Leopard, classified as Panthera uncia, is a
as All India Radio (Akashvani) and Doordarshan
large long-haired Asian cat belonging to the family
during elections.
Felidae.
○ This facility is available to national and
● The Snow Leopard is known to inhabit the mountains
recognised State parties.
of central Asia and the Indian subcontinent.
● Statement 2 is correct, The parties are allocated
○ In India, their geographical range encompasses
a base time and additional slots based on
a large part of the western Himalayas as well
performance in previous polls.
as the Eastern Himalayas.
● Statement 3 is not correct, The facility to provide
● Snow Leopards are known to have evolved to
free airtime for political parties during elections
survive in some of the harshest conditions on
was given statutory backing with the enactment of
Earth.
the 2003 amendment to the Representation of
People Act, 1951.
MAY - 2023 32
79. Explanation: 83. Explanation:
● LSD is a viral disease caused by the Capripox virus Herschel Island is an island in the Beaufort Sea (part
of the Poxviridae family which is also called the of the Arctic Ocean), which lies 5 km off the coast of
Neethling virus. Yukon in Canada, of which it is administratively a part.
● It causes prolonged morbidity in bovines such as ● It is in news as a study reveals that the thawing
cattle and buffaloes. permafrost in Herschel Island is likely to
● LSD spreads through blood-feeding insects like destabilise thousands of industrial sites and linked
flies, mosquitoes and ticks. contaminated areas in the Arctic, which could
result in the spread of toxic substances across the
region.
80. Explanation:
● The right to vote in public elections is not listed
84. Explanation:
under the Fundamental duties in the Constitution
of India. ● Statement 01 is incorrect, The right to protest
is under article 19(1) (a), article 19(1)(b) and Article
● The fundamental duties were added by the 42nd
19(1)(c) which gives the citizen the right to freedom
Amendment Act of the Constitution in 1976.
of expression and the right to meet peacefully
● The Fundamental Duties are dealt with in Article without weapons and the right to form associations
51A under Part-IV A of the Indian Constitution. or trade unions.
● Statement 02 is incorrect, The right to peaceful
81. Explanation: protest is included in the Universal Declaration
● Statement 01 is correct, Alternative Investment of Human Rights of the United Nations. The right
Funds (AIF) are any privately pooled investment of peaceful assembly includes the right to hold
fund (whether from Indian or foreign sources) in meetings, sit-ins, strikes, rallies, events or protests,
the form of a trust, a company, a body corporate, both offline and online.
or a Limited Liability Partnership, as defined by the
Securities and Exchange Board of India (Alternative 85. Explanation:
Investment Funds) Regulations, 2012.
Brahmani river originates in Jharkhand and is known
● Statement 02 is incorrect, AIF does not contain by the name of Burhabalang in Odisha. Baitarani river
funds that are subject to the SEBI’s laws. Family originates in Odisha. Bhitarkanika Wildlife Sanctuary is
trusts, employee welfare trusts, and gratuity trusts situated at the delta of Brahmani and Baitarani rivers
are not counted as AIFs. before they merge into the Bay of Bengal.
● Statement 03 is incorrect, AIFs which do not
undertake leverage or borrowing other than to 86. Explanation:
meet day-to-day operational requirements and
as permitted in the SEBI (Alternative Investment ● Statement 1 is correct, On August 6, 1945,
Funds) Regulations, 2012. Various types of funds Hiroshima became the first city in the world to be
such as real estate funds, private equity funds hit by a nuclear weapon.
(PE funds), funds for distressed assets, etc. are ● Statement 2 is correct, The Potsdam Declaration
registered as Category II AIFs. was an ultimatum issued by the U.S., Great Britain,
and China in July 1945, calling for the unconditional
surrender of Japan.
82. Explanation:
○ This declaration was made at the Potsdam
● Statement 01 is incorrect, Angel tax is a term used Conference towards the end of the World War
to refer to the income tax payable on capital raised
II.
by unlisted companies via the issue of shares
where the share price is seen in excess of the fair
market value of the shares sold. 87. Explanation:
○ The excess realisation is treated as income ● Statement 1 is not correct, The Scheduled Castes
and taxed accordingly. and the Scheduled Tribes (Prevention of Atrocities)
● Statement 02 is correct, Criteria that are Amendment Act, 2018, inserted Section 18 A to that
considered for DPIIT startup recognition are, original that nullifies the conduct of a preliminary
enquiry before registration of an FIR, or to seek
approval of any authority prior to arrest of an
accused.
● Statement 2 is correct, Further, an individual
accused of atrocities against SC ST people will be
excluded from the provision of anticipatory bail.

88. Explanation:
● The third schedule of the Indian Constitution
contains the forms of oath and affirmation for:
Source: Startupindia.gov ○ Union Ministers of India
○ Parliament Election Candidates
○ Members of Parliament (MPs)
MAY - 2023 33
○ Supreme Court Judges 92. Explanation:
○ Comptroller and Auditor General ● A person must meet the following eligibility criteria
○ State Ministers to get an OCI Card:
○ State Legislature Elections’ Candidates ○ He/She is a citizen of another country having
○ State Legislature Members an Indian origin. He/She was a citizen of
India on or before the commencement of the
○ High Court Judges Constitution; or
● The third schedule of the Indian Constitution ○ He/She is a citizen of another country, but was
does not contain oaths and affirmations for the eligible for the citizenship of India at the time
President of India and Governors of States. of the commencement of the Constitution; or
○ He/She is a citizen of another country and
89. Explanation: belonging to a territory that became a part of
● Statement 1 is correct, The post of a deputy chief India after 15th August 1947; or
minister is not a constitutional post. However, it is ○ He/She is a child/grandchild/great-grandchild
equivalent to the rank of a cabinet minister and of such a citizen; or
enjoys the perks that a cabinet minister is entitled
○ He/She is a minor child, whose parents are
to.
both Indian citizens or one parent is a citizen
● Statement 2 is not correct, The post of a deputy of India and is a spouse of foreign origin of an
chief minister does not hold any more significance Indian citizen or of an OCI cardholder.
than that of a Cabinet minister in the state.
● Furthermore:
○ The deputy chief minister’s actual importance ○ Any person having citizenship of Bangladesh
and functions depends on the portfolio that is
or Pakistan is not eligible to apply for the OCI
allocated.
card.
○ The official files that are meant for CM are not ○ Even a person having a background of
routed through the deputy CM, instead the
serving in any foreign military is also not
files pertaining to the portfolios that the deputy
eligible to apply for the OCI card.
CM gets, are routed to the CM, through him.
● Thus, Option a is the correct answer.
● Statement 3 is not correct, According to the Indian
Constitution “the Chief Minister shall be appointed
by the Governor and the other Ministers shall be 93. Explanation:
appointed by the Governor on the advice of the ● Statement 1 is correct, The International Day for
Chief Minister”. Biological Diversity is celebrated on May 22nd
○ Thus the deputy chief minister is also appointed every year.
by the Governor. ○ It aims to increase understanding and
awareness of biodiversity issues.
90. Explanation: ● Statement 2 is correct, The theme of the
● All the Acts mentioned have relevance to/bearing International Day for Biological Diversity 2023 is
on the biodiversity conservation in the country. “From agreement to action: Build back biodiversity”.
● Statement 3 is correct, India passed the Biological
Diversity Act in 2002 in order to protect biodiversity
91. Explanation:
and facilitate the sustainable management of
● Pair 1 is correct, Though Yoga was practised in biological resources with the local communities.
the pre-Vedic period, the great Sage Maharshi
Patanjali systematized and codified the then-
○ The main objective of the Act is to ensure the
conservation of biological diversity, sustainable
existing practices of Yoga, its meaning and its
use of its components and fair usage of its
related knowledge through his Yoga Sutras.
resources in order to prevent overuse or
○ Yoga school introduces the methods of the eventual destruction of biodiversity.
discipline of body and mind.
● Pair 2 is not correct, Vaisheshika school was 94. Explanation:
founded by the sage Kanada.
● The General Data Protection Regulation (GDPR) is
○ Vaisheshika School deals with metaphysics. an EU law on data protection and privacy in the
● Pair 3 is correct, Sankhya is one of the oldest European Union (EU) and the European Economic
philosophies and was put forth by the sage Kapila. Area (EEA).
● Pair 4 is correct, Sage Gautama is the founder of ● It was approved by the EU in April 2016 and came
the Nyaya school. into force on 25th May 2018.
○ Nyaya School follows a scientific and rational
approach. 95. Explanation:
● Statement 1 is not correct, The Indian Himalayan
Region (IHR) is the section of the Himalayas within
India, spanning the States and UTs of Jammu &
Kashmir, Ladakh, Himachal Pradesh, Uttarakhand,
Sikkim, Arunachal Pradesh, as well as the hill
regions of Assam and West Bengal.
MAY - 2023 34
● Statement 2 is not correct, The Western Ghats 99. Explanation:
traverse the States of Kerala, Tamil Nadu, ● Statement 1 is not correct, The National Population
Karnataka, Goa, Maharashtra and Gujarat. Register (NPR) was first prepared in 2010 and
● Statement 3 is correct, Pulicat Lake (formerly updated in 2015.
Pralaya Kaveri) straddles the border of Andhra ● Statement 2 is correct, The NPR was prepared
Pradesh and Tamil Nadu states with over 40% of it under Sub-rule (4) of Rule 3 of the Citizenship
in Andhra Pradesh and 60% in Tamil Nadu. (Registration of Citizens and Issue of National
○ It is the second largest brackish water lake or Identity Cards) Rules, 2003, framed under the
lagoon in India, after Chilika Lake. Citizenship Act, 1955.
○ The lake encompasses the Pulicat Lake Bird ● Statement 3 is not correct, The objective of the
Sanctuary. NPR is to create a comprehensive database of
○ The barrier island of Sriharikota separates the the total population residing in India and the
lake from the Bay of Bengal. total population includes both citizens and non-
citizens.
96. Explanation: ○ The category of non-citizens isn’t restricted to
illegal migrants alone.
● Statement 1 is correct, The Krishna River originates
in the Western Ghats near Mahabaleshwar in the
state of Maharashtra. 100.Explanation:
● Statement 2 is correct, The Krishna River is the ● Statement 1 is correct, Swadeshi Movement
third-longest river in India, after the Ganga and the focused on self-reliance thus contributing to the
Godavari. revival of indigenous artisan craft and industries.
○ The Krishna River is also the fourth-largest in ● Statement 2 is correct, The National Council of
terms of water inflows and river basin area in Education was established by Satish Chandra
India, after the Ganga, Indus and Godavari. Mukherjee and other Indian nationalists in Bengal
in 1906 to promote science and technology as part
● Statement 3 is correct, The Krishna River flows
of the Swadeshi industrialisation movement.
through Maharashtra, Karnataka, Andhra Pradesh,
and Telangana.
● Statement 4 is correct, Bhima River is a major left- 101. Explanation:
bank tributary of the Krishna River. ● Statement 1 is not correct, 5G technology offers
an extremely low latency rate (i.e. the delay
between the sending and receiving information) as
97. Explanation:
compared to 4G technology.
● Statement 1 is correct, Central Adoption Resource
○ The data bandwidth of 5G is significantly
Authority (CARA) is a statutory body working under
higher as compared to 4G technology.
the Ministry of Women & Child Development.
● Statement 2 is correct, The Prime Minister of India
● Statement 2 is correct, Child Adoption Resource
launched the 5G services in Oct 2022 and within
Information & Guidance System (CARINGS) is an
8 months of launch, 2,00,000 sites covering 700
online platform designed to bring transparency
districts have been installed.
to the adoption system and also curtail delays at
various levels. ○ The 5G network has been rolled out in all 28
states and 8 UTs to date.
○ The CARINGS portal for adoption is maintained
by CARA. ○ This is one of the fastest 5G rollouts in the
world.
● Statement 3 is correct, The Juvenile Justice Act
streamlines all adoption and child safety-related
rules in the country. 102. Explanation:
● Statement 1 is correct, Article 371 C of the Indian
98. Explanation: Constitution deals with the special provision with
respect to the State of Manipur.
● Statement 1 is correct, Article 123 of the
Constitution grants the President certain law- ● Statement 2 is correct, According to Article 371
making powers to promulgate ordinances during C, the President is authorised to provide for the
the recess of Parliament. creation of a committee of the Manipur Legislative
Assembly consisting of members elected from the
● Statement 2 is correct, Article 213 deals with the
Hill Areas of the State.
power of the Governor to promulgate ordinances.
○ In this article, the expression ‘Hill Areas’ means
● Statement 3 is not correct, The maximum life of an
such areas as the President may, by order,
ordinance can be six months and six weeks.
declare to be Hill Areas.
○ Ordinances passed are to be presented
● Statement 3 is not correct, Manipur hills are a
before the Parliament/State legislatures upon
part of the Purvanchal Mountains or the Eastern
reassembling.
Highlands.
○ Such ordinances must be approved by
○ The Purvanchal includes the Patkai Hills, Naga
Parliament/State legislatures within six weeks
Hills, Mizo Hills and Manipur Hills.
of reassembling or they shall cease to operate.
MAY - 2023 35
103. Explanation: important part in the war of 1971.
● Statement 1 is not correct, Central Board of Film ● Statement 3 is correct, INS Vikramaditya is the
Certification (CBFC) is a statutory body under the modernised and refurbished version of an original
Ministry of Information and Broadcasting. Kiev class aircraft carrier.
○ It is mandated with the task of regulating the ○ Post the dissolution of the USSR, it was
public exhibition of films under the provisions rechristened Admiral Gorshkov.
of the Cinematograph Act 1952. ● Statement 4 is correct, INS Viraat was originally
● Statement 2 is correct, Films can be publicly commissioned by the British Royal Navy as HMS
exhibited in India only after they have been Hermes in November 1959.
certified by the Central Board of Film Certification
(CBFC). 108. Explanation:
● Statement 3 is correct, CBFC assigns each film to ● Statement 1 is not correct, X-Ray Polarimeter
one of four categories. The categories are: Satellite (XPoSat) ) is India’s first dedicated
polarimetry mission to study various dynamics
of bright astronomical X-ray sources in extreme
conditions.
○ ISRO is collaborating with the Raman Research
Institute (RRI) to build XPoSat.
Source: CBFC ● Statement 2 is correct, XXPoSat aims to study the
dynamics of the bright astronomical X-Ray sources
104. Explanation: under extreme conditions.
● Statement 1 is correct, The first World Conference ○ It has two payloads namely X-Ray Polarimeter
on Disaster Risk Reduction was held in Yokohama, named POLIX and X-ray SPECtroscopy and
Japan in 1994. Timing identified as XSPECT.
● Statement 2 is correct, Priority Four of the Sendai
Framework notes that the recovery, rehabilitation 109. Explanation:
and reconstruction phase is a “critical opportunity ● Pair 1 is not correct, Bakhmut is a city in eastern
to Build Back Better.’” Ukraine.
● Statement 3 is not correct, The Hyogo Framework ○ It is an administrative centre and an industrial
of Action was adopted in 2005 at the World hub in the Bakhmut Raion of Donetsk region.
Conference on Disaster Reduction held in Kobe,
Hyogo, Japan. ● Pair 2 is correct, Gaziantep which is still informally
called Antep, is a major city in south-central Turkey.
○ It is the capital of the Gaziantep Province and
105. Explanation:
the sixth-most populous city in Turkey.
● The Consumer Price Index (CPI) for industrial ● Pair 3 is correct, Nusantara is the future capital of
workers and agricultural labourers are published
Indonesia scheduled to be inaugurated in 2024.
by the Labour Bureau.
○ Nusantara is located on the east coast of
● The CPI for industrial workers measures the Borneo, which is the world’s third-largest
average change in retail prices of a basket of
island.
commodities which an industrial worker generally
consumes.
● The CPI for industrial workers is increasingly being 110. Explanation:
considered the appropriate indicator of general ● The term ‘paramita’ is commonly translated as
inflation, which shows the most accurate impact of “perfection“.
price rise on the cost of living of common people. ● According to Buddhism, a pilgrim’s path to
● The items included in CPI for industrial workers becoming a Samma Sambuddha (Enlightened
are food, pan, supari, tobacco, fuel and lighting, Universal Buddha) or Bodhisattva consists of
housing, clothing, and miscellaneous expenses perfecting certain qualities.
with food being accorded the highest weight. ● In Southern traditions, these qualities are known
as Paramis (perfections), while in Eastern and
106. Explanation: Northern traditions, they are known as Paramitas.
● Project Shivalik is a task force of the Border Roads ● The Ten Paramitas are – Generosity, Morality,
Organisation (BRO) that maintains more roads in Patience, Energy, Meditation, Wisdom, Skilful
Uttarakhand, including national highways along means, Resolution, Power, and Knowledge.
the pilgrimage trail.
111. Explanation:
107. Explanation: ● Statement 1 is correct: CBAM is introduced by
● Statement 1 is correct, INS Vikrant was the Indian the EU to reduce carbon emissions by imposing
Navy’s first aircraft carrier inducted into service. a carbon tax on imported products, ensuring that
● Statement 2 is correct, India’s first-ever Indigenous they are subject to the same carbon costs as
Aircraft Carrier (IAC) – INS Vikrant was named in products produced within the EU.
honour of the historic predecessor that played an
MAY - 2023 36
● Statement 2 is correct: It is part of the EU’s “Fit for ● Statement 3 is correct, Bureau of Indian Standards
55 in 2030 package” to reduce greenhouse gas (BIS) is a founder member of ISO. BIS represents
emissions by at least 55% by 2030 compared to India in the ISO.
1990 levels.
117. Explanation:
112. Explanation: ● BIS is a founding member of International
● Indian Strategic Petroleum Reserve Limited (ISPRL), Organization for Standardization (ISO) and it joined
a Government of India Special Purpose Vehicle, has the International Electrotechnical Commission
established Strategic Petroleum Reserves (SPR) (IEC) in 1947.
facilities at 3 locations, namely Vishakhapatnam, ● BIS is also a member of regional standards bodies
Mangaluru, and Padur. like Pacific Area Standards Congress (PASC), South
Asian Regional Standards Organization (SARSO),
113. Explanation: World Standards Service Network (WSSN) and
● Statement 1 is not correct: The UPSC comprises under the framework of IBSA (India, Brazil and
of a chairman and ten members. The President South Africa).
of India appoints the UPSC Chairman and other 118. Explanation:
members. Each member holds office for a tenure ● Statement 1 is correct, Sagar Parikrama is an
of 6 years or till he becomes the age of 65 years. initiative taken by Government of India, with an
● Statement 2 is not correct: UPSC is not consulted aim to resolve the issues of the fishers, other
for making reservations of post in favor of backward stakeholders and facilitate their economic
classes. upliftment through various fisheries schemes and
● Statement 3 is not correct: Any person who has programs implemented by the Government of
once held the office as a member of a Public Service India such as Pradhan Mantri Matsya Sampada
Commission is ineligible for reappointment to that Yojana (PMMSY) and Kisan Credit Card (KCC).
office. However, they are eligible for appointment ● Statement 2 is not correct, Phase-I of the
as the Chairman or any other member of the UPSC programme Sagar Parikrama was organised in
or SPSC. Gujarat in March 2022 from Mandvi and ended on
at Porbandar, Gujarat.
114. Explanation: ○ Phase –II programme started in September
2022 from Mangrol to Veraval, Mul Dwaraka,
● Statement 1 is correct: XPoSat will study various
and finally ended at Madhwad.
dynamics of bright astronomical X-ray sources in
extreme conditions. ○ Phase-III programme started in Feb 2023 from
Surat, Gujarat, and ended at Sasson Dock,
● Statement 2 is correct: It is India’s first, and only the
Mumbai.
world’s second polarimetry mission that is meant
to study various dynamics of bright astronomical ○ Phase-IV programme started from Mormugao
X-ray sources in extreme conditions. Port, Goa in March 2023 and ended in
Mangalore.
115. Explanation: ● Statement 3 is correct, India is the third largest
fish producing country which accounts for 8% of
● Statement 1 is correct: Micro-irrigation helps in the global fish production.
reducing fertilizer and nutrient loss.
○ India also ranks second in aquaculture
● Statement 2 is incorrect: It is not the only means production.
of irrigation in dry land farming.
● Statement 3 is correct: It helps in reducing
119. Explanation:
groundwater depletion.
● According to Article 164 (1A) of the Constitution,
the total number of Ministers, including the Chief
116. Explanation: Minister, in the Council of Ministers in a State shall
● Statement 1 is not correct, The International not exceed fifteen per cent. of the total number of
Organization for Standardization (ISO) develops members of the Legislative Assembly of that State:
and publishes standardization in all technical ○ Provided that the number of Ministers,
and nontechnical fields other than electrical and including the Chief Minister in a State shall not
electronic engineering. be less than twelve.
○ The standardization of electrical and electronic ● This clause was introduced/added through the
engineering is handled by the International 91st Constitutional Amendment Act of 2003.
Electrotechnical Commission (IEC).
● Hence “Statement—I is incorrect but Statement—
● Statement 2 is not correct, ISO is an independent, II is correct”.
non-governmental international organization
with a membership of 168 national standards
bodies.
○ The ISO has its headquarters in Geneva,
Switzerland.
MAY - 2023 37
120. Explanation: 123. Explanation:
● The following countries have their own navigation ● Protection Of Children from Sexual Offences Act
satellite system. (POCSO) was passed by the parliament in the year
○ The United States: GPS – World’s most 2012 with an intent to effectively address the evil of
used GPS system, operational from 1978. sexual exploitation and sexual abuse of children.
Constellation of 32 satellites. ● The act aims at making offences against children
○ Russia: GLONASS – Constellation of 24 gender-neutral and sets a gender-neutral tone for
satellites. the legal framework available to child sexual abuse
victims.
○ European Union: Galileo – Became operational
in 2016, with a constellation of 30 satellites. ● Thus, the act also does not distinguish between
perpetrators of child sexual abuse on the grounds
○ China: BeiDou – Currently it provides regional
of gender, and there have been instances where
coverage of the Asia- Pacific region, plans to
the courts have convicted women for such abuse.
provide global coverage by 2020.
○ Japan: Quasi-Zenith Satellite System (QZSS) –
It is a regional satellite system covering Japan 124. Explanation:
and Asia-Oceania region. ● Statement 1 is not correct, According to Article
○ India: NAVIC or The Indian Regional Navigation 148 (6) of the Constitution, the administrative
Satellite System. expenses of the office of the Comptroller and
Auditor General, including all salaries, allowances
and pensions payable to or in respect of persons
121. Explanation: serving in that office, shall be charged upon the
● The Representation of the People Act, 1950 include Consolidated Fund of India.
the provisions for: ● Statement 2 is not correct, The reports of the
○ Seat allocation in the Lok Sabha and the Comptroller and Auditor-General of India relating
Legislative Assemblies through direct to the accounts of the Union namely audit report
elections. on appropriation accounts, audit report on finance
○ The voters’ qualifications for the elections. accounts, and audit report on public undertakings,
○ The delimitation of constituencies for both Lok are to be submitted to the President.
Sabha and Assembly elections. The extent of ○ President shall cause them to be laid before
the constituencies would be determined by each House of Parliament.
the Delimitation Commission.
○ The Indian President can alter the 125. Explanation:
constituencies after due consultation with the ● Salinisation occurs when the irrigation water
Election Commission. accumulated in the soil evaporates, leaving behind
○ Preparation of the electoral roll. A person salts and minerals.
cannot be enrolled for over one constituency. ● Salinisation makes some soils of the irrigated land
He or she can be disqualified and barred from impermeable.
voting if found to be of unsound mind or is not
● Excess soil salinity, therefore, causes poor and
an Indian citizen.
spotty stands of crops, uneven and stunted growth
● Section 39A of the Representation of the People and poor yields.
Act, 1951 deals with the “Allocation of equitable
sharing of time”.
126. Explanation:
● Section 78A of the Representation of the People
Act, 1951 deals with the “Free supply of copies of ● Statement 1 is not correct, Goa was annexed by
electoral rolls. India on 19 December 1961.
○ On 30 May 1987 Goa attained statehood
(while Daman and Diu became a separate
122. Explanation:
union territory).
● Statement 1 is correct, The Official Secrets Act of ● Statement 2 is correct, In 1492, Goa became a part
1923 is India’s anti-espionage.
of Adil Shah’s Bijapur Sultanate, which established
○ The Official Secrets Act, 1923 extends to the Goa Velha as its second capital.
whole of India and applies also to servants
○ Goa was captured by the Portuguese in the
of the Government and to citizens of India
Battle of Goa (1510) and became the first part
outside India.
of India to fall to European colonial rule on the
● Statement 2 is not correct, Lahore Conspiracy west coast.
Case trial was held by a Special tribunal constituted
● Statement 3 is correct, Operation Vijay (1961) is
under the Defence of India Act 1915.
the operation by the Military of India that led to
● Statement 3 is correct, The Second Administrative the capture of Goa, Daman and Diu and Anjediva
Reforms Commission (ARC), in its Report of June Islands from the Portuguese.
2006 had recommended that the Official Secrets
Act (OSA) of 1923 should be repealed, and
substituted by a chapter in the National Security
Act, containing provisions relating to official
secrets.
MAY - 2023 38
127. Explanation: 129. Explanation:
● Polygenic risk scores can provide a measure of
disease risk due to one’s genes.
● It summarises genome-wide genotype data into a
single number that represents genetic liability to a
trait.
● Polygenic risk score is one way by which people
can learn about their risk of developing a disease,
based on the total number of changes related to
the disease.

130. Explanation:
● The Supreme Court of India in its Samatha vs
State of A.P. and Ors. case held that government,
tribal, and forested lands in the scheduled areas
cannot be leased to non-tribal persons or private
companies for mining purposes.
128. Explanation: ● The court noted that the Fifth Schedule of the
Indian Constitution preserved these lands to
● Statement 1 is correct, Flavonols occur abundantly
protect tribal persons’ economic empowerment,
in a variety of fruits and vegetables.
economic justice, social status, and dignity.
○ The most studied flavonols are kaempferol,
● The Court also noted that all land in the scheduled
quercetin, myricetin and fisetin
areas, regardless of title, cannot be leased out
● Statement 2 is correct, Flavonols act in plants because of the importance of agriculture as the
as antioxidants, antimicrobials, photoreceptors, source of livelihood for tribal persons and thus the
visual attractors, feeding repellants, and for light transfer of lands in the scheduled areas can be
screening. allowed only for peace and good governance of
● Statement 3 is correct, Flavonols deficiency in the land.
humans is associated with age-related memory
loss.
MAY - 2023 39

UPSC MAINS PRACTICE QUESTIONS


1. India’s poor waste management is at the root of multiple 13. There is an urgent need to replace the colonial era
urban issues in our cities. Elaborate. Drugs and Cosmetics Act 1940. Justify.
(250 Words; 15 Marks) (GS-3; Environment) (10 Marks, 150 Words) (GS-2; Health)

2. What are Sustainable Development Goals? Evaluate 14. Reservation of electoral seats is an effective tool to
the progress made by India in achieving the SDGs. safeguard the interest of the weaker groups. Comment.
(250 Words; 15 Marks) (GS-3; Environment) (15 Marks, 250 Words) (GS-2; Social Justice)

3. Gig and platform workers are deprived of regular 15. Europe’s idea of fighting climate change through the
worker and social security benefits. Discuss. Carbon Border Adjustment Mechanism is discriminatory
(15 Marks, 250 Words) [GS-3, Economy] against developing nations. Do you agree? Elaborate.
(250 Words; 15 Marks) (GS-3; Environment)
4. What do you understand by de-dollarisation? Why has
the US Dollar remained the preferred global reserve 16. More than the NRIs, it is the domestic migrant labourers
currency for such a long time? Does any other currency that deserve the right to remote voting. In this context,
have the potential to replace the US dollar? discuss the initiatives taken by the Election Commission
(15 Marks, 250 Words) [GS-3, Economy] of India in this regard.
(250 Words; 15 Marks) (GS-2; Polity)
5. Why has India been classified as a laundromat country?
Explain. 17. How can India’s new Space Policy ensure more
(15 Marks, 250 Words) [GS-2, International Relations] active participation from the private sector in Space?
Elaborate.
6. Evaluate the need to regulate AI and analyse the global (250 Words; 15 Marks) [GS-3; Science and Technology]
trends in AI regulation.
(15 Marks, 250 Words) [GS-3, Science & Technology] 18. Why is the Indian Civil Aviation sector considered
a deathbed for so many airlines? List out the major
reasons ailing the Indian Airlines industry.
7. Despite the promise it holds, India’s civil aviation sector
has been troubled for long. Elucidate. (250 Words; 15 Marks) [GS-3; Economy]
(250 Words, 15 Marks) [GS-3; Economy]
19. How would China’s new policy of playing a mediator in
international conflicts impact India? Discuss.
8. Examine the strategic implications of the ‘Washington
Declaration’ that was signed recently between the USA (250 Words; 15 Marks) (GS-2; International Relations)
& South Korea.
(250 Words, 15 Marks) [GS-2; International Relations] 20. Anti–defection law takes away the right of the
legislators to express their true opinion. Do you agree?
Critically analyze this statement.
9. Discuss the challenges which India faces as the
president of the G20 and the leader of the SCO. (250 Words; 15 Marks) (GS-2; Polity)
(150 Words, 10 Marks) (GS-2; International Relations)
21. “Besides being a moral imperative of a Welfare State,
primary health structure is a necessary precondition for
10. “Smart Metering is crucial for India’s electricity sustainable development.” Analyze.
revolution”. Comment.
(250 Words; 15 Marks) (GS-2;Governance)
(150 Words, 10 Marks) (GS-3; Economy)

22. No Right to Health legislation should come at the cost


11. On Taliban, India and China share similar concerns of medical practitioners. In this context, explain the
and, therefore, must engage with each other for mutual issues raised by doctors against Rajasthan’s Right to
benefit and regional peace. Comment. Health Act.
(150 Words; 10 Marks) (GS-2; International Relations) (250 Words; 15 Marks) (GS-2;Governance)

12. Restrictions to freedom of expression in India are often 23. What do you mean by a 3-parent baby? Explain the
cast as a colonialera hangover but they are also rooted step by step process used in such procedures.
in the popular feelings of society and are in a sense,
reflective of local norms. Comment. (250 Words; 15 Marks) (GS-3; Science and Technology)
(250 Words; 15 Marks) (GS-2; Polity and Governance)
MAY - 2023 40

24. Slowly but surely, India is getting rid of its 36. What is Open Network for Digital Commerce (ONDC)?
reluctance to join multiparty economic agreements. How is it the next revolution in the e-commerce sector?
Elaborate with appropriate examples. (150 Words, 10 Marks) (GS-2; Governance)
(250 Words; 15 Marks) (GS-3; Economy)
37. Education is not an injunction, it is an effective and
25. The well-established convention of ‘Bail is rule & Jail is pervasive tool for the all-round development of an
an exception’ is hardly followed at the trial court level. individual and social transformation. Examine the New
Do you agree? Critically analyze the reasons in support Education Policy, 2020 (NEP, 2020) in light of the
of your answer. above statement.
(250 Words; 15 Marks) (GS-2; Social Justice) (250 Words; 15 Marks) (GS-2; Education)

26. The PoSH Act, 2013 is a classic example of a good 38. Despite their smaller size, the Pacific Island nations
law made ineffective due to faulty implementation. hold great significance for India’s global ambitions.
Elaborate with appropriate examples. Comment.
(250 Words; 15 Marks) (GS-2; Polity) (250 Words; 15 Marks) (GS-2; International Relations)

27. Despite several attempts, the banning of liquor hasn’t 39. India and Australia are natural strategic partners.
been successful in most Indian states. Do you think Analyse the statement.
that regulating liquor would give us a better chance (15 Marks, 250 Words) [GS-2, IR]
to avoid Hooch tragedies as compared to banning it
completely? Critically analyze the reasons in support of
your answer. 40. BRICS grouping has lost its relevance. Do you agree?
Critically discuss.
(250 Words; 15 Marks) (GS-2; Health)
(15 Marks, 250 Words) [GS-2, IR]

28. Lower imports may seem like a good way to cut


down trade deficits, but it comes with its own set of 41. Evaluate the recent shifts in the geopolitics of the Arab
challenges. Elaborate. world, particularly under the leadership of Saudi Arabia.
What implications does it have for Indian interests in
(250 Words; 15 Marks) (GS-3; Economy) West Asia?
(250 Words, 15 Marks) (GS-2; International Relations)
29. Palliative care is an often ignored part of the public
healthcare system. Do you agree? What steps have
been taken by the government to change this situation? 42. How does the emergence of generative AI impact
copyright laws? Analyse.
(250 Words; 15 Marks) (GS-2; Health)
(250 Words, 15 Marks) (GS-3; Science and Technology)

30. Has the time come to take a long look at the anti-
defection law and bring it in line with the right to 43. Examine the constitutionality of The Government
freedom of expression for the legislators in India? of National Capital Territory of Delhi (Amendment)
Discuss. Ordinance, 2023.
(250 Words; 15 Marks) (GS-2; Polity) (250 Words, 15 Marks) (GS-2; Polity)

31. “India is walking a tightrope diplomatically in the 44. Critically analyze the trickle-down model of economic
current geopolitics”. Comment. development. Does it provide for universal social and
economic well-being?
(150 Words, 10 Marks) (GS-2; International Relations)
(250 Words, 15 Marks) (GS-3; Economy)

32. What are green deposits? What are the RBI regulations
with regard to them? 45. Acts of sedition can be serious internal security threat
to the country. However, the very concept of sedition
(150 Words, 10 Marks) (GS-3; Economy) needs a closer review with respect to its interpretation,
scope and misuse by the state. Discuss
33. Evaluate India’s progress on various Sustainable (250 Words; 15 Marks) [GS-2; Polity]
Development Goals. Identify some of the initiatives
taken by the Government in this regard.
46. Apart from being used to demarcate constituencies,
(250 Words; 15 Marks) (GS-2; Governance) updated decadal census data is vital to administration.
How will the indefinite postponement of census impact
34. Green Hydrogen may hold the key to solving India’s the policy formation for the future? Critically Analyse.
renewable energy issues. Discuss. (250 Words; 15 Marks) [GS-2; Polity and Governance]
(250 Words; 15 Marks) (GS-3; Environment)
47. The ongoing US – China chip war has significant
35. What is bio-manufacturing and how can India harness implications on India as well. Discuss.
its potentialities? (250 Words; 15 Marks) (GS-2; International
(150 Words, 10 Marks) (GS-3; Economy) Relations)
MAY - 2023 41

48. Despite multiple conferences and summits, the world


is all set to breach the 1.5 degree temperature increase
mark. Identify the reasons for this collective failure.
(250 Words; 15 Marks) (GS-3; Environment)

49. India has made a lot of progress in the world of


supercomputers and yet lags behind its peers. Critically
analyse.
(250 Words, 15 Marks) (GS-3; Science and
Technology)

50. Examine the powers of Chief Justice of India in the


context of him/her being the Master of Roster and
concerns surrounding these powers.
(250 Words, 15 Marks) (GS-2; Polity)

51. Explain the relevance of soft power diplomacy in India’s


foreign policy with examples.
(250 Words, 15 Marks) (GS-2; International
Relations)

52. Deduce the US policy shift from de-coupling to de-


risking with China.
(250 Words, 15 Marks) (GS-2; International
Relations)

You might also like